Dataset Preview
Full Screen
The full dataset viewer is not available (click to read why). Only showing a preview of the rows.
The dataset generation failed because of a cast error
Error code:   DatasetGenerationCastError
Exception:    DatasetGenerationCastError
Message:      An error occurred while generating the dataset

All the data files must have the same columns, but at some point there are 22 new columns ({'Solution_2_.28Simple.29', 'Solution_with_Explanations', 'Solution_where', 'Solution_4_Without_the_nasty_computations', 'Solution_.28No_Bash.29', 'Solution_3_.28Angle-Chasing.29', 'Solution_Number_Sense', 'Solution_3_.28INCORRECT.29', 'Solution_2_.28Taken_from_Twitch_Solves_ISL.29', 'Solution_3:_Graph_Theoretic', 'Problem_4', 'Solution_3_.28Less_technical_bary.29', 'Solution_with_Thought_Process', 'Solution_2_.28and_motivation.29', 'Solution_3_.28Not_a_formal_proof_but_understandable.29', 'Solution_5.28stupid_but_simple.29', 'Solution_4_.28Less_bashy_bary.29', 'Solution_3_.28without_induction.29', 'Solution_3_.28clear_Solution_2.29', 'Solution_6_.28Motivation_for_Solution.29', 'Solution_1:_Pigeonhole', 'Solution_2.2C_Barycentric_.28Modified_by_Evan_Chen.29'}) and 26 missing columns ({'Solution_2_2', 'Solution4', 'Solution_3_.28Visual.29', 'Solution_7_from_Brilliant_Wiki_.28Muirheads.29_.3D', 'Solution_2_.28duality_principle.29', 'Solution_7', 'Problem_2', 'Solution_2.28SFFT.29', 'Solution_2_.28Avoids_the_error_of_the_first.29', 'Solution_3_.28No_Miquel.27s_point.29', 'Problems', 'Solution_2_.28Sort_of_Root_Jumping.29', 'Solution_1_2', 'Solution_3_.28Complex_Bash.29', 'Solution1', 'Solution_2_.28Strong_Induction_and_Alternative_Case_Division.29', 'Solution_3:_The_only_one_that_actually_works', 'Solution_4:_This_works_as_well', 'Solution_2_.28elegant.29', 'Solution_1.1', 'Solution_3b', 'Solution_3_.28Trigonometry.29', 'Solution_8_.28fast_Titu.27s_Lemma_no_substitutions.29', 'Solution_1_.28Induction_and_Case_Division.29', 'Solution_1_.28Efficient.29', 'Solution_2_.28Three_perpendicular_bisectors.29'}).

This happened while the json dataset builder was generating data using

hf://datasets/autores/imo_hq/usajmo_hq.json (at revision 2e0d2a2e692e42e4ab668040208ca4d9e95fe337)

Please either edit the data files to have matching columns, or separate them into different configurations (see docs at https://hf.co/docs/hub/datasets-manual-configuration#multiple-configurations)
Traceback:    Traceback (most recent call last):
                File "/src/services/worker/.venv/lib/python3.9/site-packages/datasets/builder.py", line 2011, in _prepare_split_single
                  writer.write_table(table)
                File "/src/services/worker/.venv/lib/python3.9/site-packages/datasets/arrow_writer.py", line 585, in write_table
                  pa_table = table_cast(pa_table, self._schema)
                File "/src/services/worker/.venv/lib/python3.9/site-packages/datasets/table.py", line 2302, in table_cast
                  return cast_table_to_schema(table, schema)
                File "/src/services/worker/.venv/lib/python3.9/site-packages/datasets/table.py", line 2256, in cast_table_to_schema
                  raise CastError(
              datasets.table.CastError: Couldn't cast
              Solution_1: string
              Solution_2_.28Simple.29: string
              Solution_with_Explanations: string
              Solution_where: string
              Solution_4_Without_the_nasty_computations: string
              Solution_2: string
              Solution_.28No_Bash.29: string
              Solution_Number_Sense: string
              Solution_3_.28Angle-Chasing.29: string
              Solution_3_.28INCORRECT.29: string
              Solution_4: string
              Solutions: string
              Solution_2_.28Taken_from_Twitch_Solves_ISL.29: string
              Solution_3:_Graph_Theoretic: string
              Problem_4: string
              Solution_3_.28Less_technical_bary.29: string
              Solution_6: string
              Solution_with_Thought_Process: string
              Solution_2_.28and_motivation.29: string
              Problem_5: string
              Solution_3_.28Not_a_formal_proof_but_understandable.29: string
              Solution_5.28stupid_but_simple.29: string
              Solution: string
              Solution_4_.28Less_bashy_bary.29: string
              Problem: string
              Solution_5: string
              Solution_3_.28without_induction.29: string
              Solution_3_.28clear_Solution_2.29: string
              Solution_6_.28Motivation_for_Solution.29: string
              Solution_1:_Pigeonhole: string
              Problem_6: string
              Solution_2.2C_Barycentric_.28Modified_by_Evan_Chen.29: string
              Solution_3: string
              to
              {'Solution_1': Value(dtype='string', id=None), 'Solution_2_2': Value(dtype='string', id=None), 'Solution4': Value(dtype='string', id=None), 'Solution_3_.28Visual.29': Value(dtype='string', id=None), 'Solution_2': Value(dtype='string', id=None), 'Solution_7_from_Brilliant_Wiki_.28Muirheads.29_.3D': Value(dtype='string', id=None), 'Solution_2_.28duality_principle.29': Value(dtype='string', id=None), 'Solution_4': Value(dtype='string', id=None), 'Solution_7': Value(dtype='string', id=None), 'Problem_2': Value(dtype='string', id=None), 'Solutions': Value(dtype='string', id=None), 'Solution_2.28SFFT.29': Value(dtype='string', id=None), 'Solution_2_.28Avoids_the_error_of_the_first.29': Value(dtype='string', id=None), 'Solution_6': Value(dtype='string', id=None), 'Solution_3_.28No_Miquel.27s_point.29': Value(dtype='string', id=None), 'Problems': Value(dtype='string', id=None), 'Solution_2_.28Sort_of_Root_Jumping.29': Value(dtype='string', id=None), 'Solution_1_2': Value(dtype='string', id=None), 'Solution_3_.28Complex_Bash.29': Value(dtype='string', id=None), 'Problem_5': Value(dtype='string', id=None), 'Solution1': Value(dtype='string', id=None), 'Solution_3': Value(dtype='string', id=None), 'Solution_2_.28Strong_Induction_and_Alternative_Case_Division.29': Value(dtype='string', id=None), 'Solution': Value(dtype='string', id=None), 'Solution_3:_The_only_one_that_actually_works': Value(dtype='string', id=None), 'Solution_4:_This_works_as_well': Value(dtype='string', id=None), 'Problem': Value(dtype='string', id=None), 'Solution_2_.28elegant.29': Value(dtype='string', id=None), 'Solution_1.1': Value(dtype='string', id=None), 'Solution_5': Value(dtype='string', id=None), 'Solution_3b': Value(dtype='string', id=None), 'Solution_3_.28Trigonometry.29': Value(dtype='string', id=None), 'Solution_8_.28fast_Titu.27s_Lemma_no_substitutions.29': Value(dtype='string', id=None), 'Solution_1_.28Induction_and_Case_Division.29': Value(dtype='string', id=None), 'Solution_1_.28Efficient.29': Value(dtype='string', id=None), 'Problem_6': Value(dtype='string', id=None), 'Solution_2_.28Three_perpendicular_bisectors.29': Value(dtype='string', id=None)}
              because column names don't match
              
              During handling of the above exception, another exception occurred:
              
              Traceback (most recent call last):
                File "/src/services/worker/src/worker/job_runners/config/parquet_and_info.py", line 1321, in compute_config_parquet_and_info_response
                  parquet_operations = convert_to_parquet(builder)
                File "/src/services/worker/src/worker/job_runners/config/parquet_and_info.py", line 935, in convert_to_parquet
                  builder.download_and_prepare(
                File "/src/services/worker/.venv/lib/python3.9/site-packages/datasets/builder.py", line 1027, in download_and_prepare
                  self._download_and_prepare(
                File "/src/services/worker/.venv/lib/python3.9/site-packages/datasets/builder.py", line 1122, in _download_and_prepare
                  self._prepare_split(split_generator, **prepare_split_kwargs)
                File "/src/services/worker/.venv/lib/python3.9/site-packages/datasets/builder.py", line 1882, in _prepare_split
                  for job_id, done, content in self._prepare_split_single(
                File "/src/services/worker/.venv/lib/python3.9/site-packages/datasets/builder.py", line 2013, in _prepare_split_single
                  raise DatasetGenerationCastError.from_cast_error(
              datasets.exceptions.DatasetGenerationCastError: An error occurred while generating the dataset
              
              All the data files must have the same columns, but at some point there are 22 new columns ({'Solution_2_.28Simple.29', 'Solution_with_Explanations', 'Solution_where', 'Solution_4_Without_the_nasty_computations', 'Solution_.28No_Bash.29', 'Solution_3_.28Angle-Chasing.29', 'Solution_Number_Sense', 'Solution_3_.28INCORRECT.29', 'Solution_2_.28Taken_from_Twitch_Solves_ISL.29', 'Solution_3:_Graph_Theoretic', 'Problem_4', 'Solution_3_.28Less_technical_bary.29', 'Solution_with_Thought_Process', 'Solution_2_.28and_motivation.29', 'Solution_3_.28Not_a_formal_proof_but_understandable.29', 'Solution_5.28stupid_but_simple.29', 'Solution_4_.28Less_bashy_bary.29', 'Solution_3_.28without_induction.29', 'Solution_3_.28clear_Solution_2.29', 'Solution_6_.28Motivation_for_Solution.29', 'Solution_1:_Pigeonhole', 'Solution_2.2C_Barycentric_.28Modified_by_Evan_Chen.29'}) and 26 missing columns ({'Solution_2_2', 'Solution4', 'Solution_3_.28Visual.29', 'Solution_7_from_Brilliant_Wiki_.28Muirheads.29_.3D', 'Solution_2_.28duality_principle.29', 'Solution_7', 'Problem_2', 'Solution_2.28SFFT.29', 'Solution_2_.28Avoids_the_error_of_the_first.29', 'Solution_3_.28No_Miquel.27s_point.29', 'Problems', 'Solution_2_.28Sort_of_Root_Jumping.29', 'Solution_1_2', 'Solution_3_.28Complex_Bash.29', 'Solution1', 'Solution_2_.28Strong_Induction_and_Alternative_Case_Division.29', 'Solution_3:_The_only_one_that_actually_works', 'Solution_4:_This_works_as_well', 'Solution_2_.28elegant.29', 'Solution_1.1', 'Solution_3b', 'Solution_3_.28Trigonometry.29', 'Solution_8_.28fast_Titu.27s_Lemma_no_substitutions.29', 'Solution_1_.28Induction_and_Case_Division.29', 'Solution_1_.28Efficient.29', 'Solution_2_.28Three_perpendicular_bisectors.29'}).
              
              This happened while the json dataset builder was generating data using
              
              hf://datasets/autores/imo_hq/usajmo_hq.json (at revision 2e0d2a2e692e42e4ab668040208ca4d9e95fe337)
              
              Please either edit the data files to have matching columns, or separate them into different configurations (see docs at https://hf.co/docs/hub/datasets-manual-configuration#multiple-configurations)

Need help to make the dataset viewer work? Make sure to review how to configure the dataset viewer, and open a discussion for direct support.

Solution_1_2
null
Solution_6
string
Solution_2_2
null
Solution_1
string
Problem_5
null
Solution_1_.28Efficient.29
null
Solution_2_.28Avoids_the_error_of_the_first.29
null
Solution_2_.28Strong_Induction_and_Alternative_Case_Division.29
null
Problem_6
null
Solution_1.1
null
Solution1
null
Solution_2_.28Sort_of_Root_Jumping.29
null
Solution_2_.28elegant.29
null
Solution_3_.28No_Miquel.27s_point.29
null
Solution4
null
Solution_2_.28Three_perpendicular_bisectors.29
null
Solution_7_from_Brilliant_Wiki_.28Muirheads.29_.3D
null
Solution_3:_The_only_one_that_actually_works
null
Solution_8_.28fast_Titu.27s_Lemma_no_substitutions.29
null
Solution_3_.28Complex_Bash.29
null
Solution_4:_This_works_as_well
null
Solution_3_.28Visual.29
null
Solution_3
string
Problems
null
Solutions
null
Solution_1_.28Induction_and_Case_Division.29
null
Solution_5
string
Problem_2
null
Solution_2_.28duality_principle.29
null
Solution_4
string
Solution_7
string
Problem
string
Solution
string
Solution_2
string
Solution_2.28SFFT.29
null
Solution_3b
null
Solution_3_.28Trigonometry.29
null
null
To understand why its irreducible, let's take a closer look at the fraction itself. If we were to separate both fractions, we end up with $\frac{21n}{14n}$ + $\frac{4}{3}$. Simplifying the fraction, we end up with. $\frac{3}{2}$. Now combining these fractions with addition as shown before in the problem, we end up with $\frac{17}{6}$. It's important to note that 17 is 1 off from being divisible by 6, and you'll see why later down this explanation. Now we expirement with some numbers. Plugging 1 in gives us $\frac{25}{17}$. Notice how both numbers are one off from being divisible by 8(25 is next to 24, 17 is next to 16). Trying 2, we end up with $\frac{46}{31}$. Again, both results are 1 off from being multiples of 15. Trying 3, we end up with $\frac{67}{45}$. Again, both end up 1 away from being multiples of 22. This is where the realization comes in that two scenarios keep reoccurring: 1. Both Numerator and Denominator keep ending up 1 value away from being multiples of the same number, but 2. One always ends up being a prime number. Knowing that prime numbers only factors are 1 and itself, the fraction ends up being a paradoxical expression where one prime number is always being produced, and even with larger values, like say 15, implementing it in gives us $\frac{319}{199}$, we keep ending up with results that at the end will only have a common divisor of 1.
null
Denoting the greatest common divisor of $a, b$ as $(a,b)$, we use the Euclidean algorithm: Their greatest common divisor is 1, so $\frac{21n+4}{14n+3}$ is irreducible. Q.E.D.
null
null
null
null
null
null
null
null
null
null
null
null
null
null
null
null
null
null
Proof by contradiction: Assume that $\dfrac{14n+3}{21n+4}$ is a reducible fraction. If a certain fraction $\dfrac{a}{b}$ is reducible, then the fraction $\dfrac{2a}{3b}$ is reducible, too. In this case, $\dfrac{2a}{3b} = \dfrac{42n+8}{42n+9}$. This fraction consists of two consecutives numbers, which never share any factor. So in this case, $\dfrac{2a}{3b}$ is irreducible, which is absurd. Hence $\frac{21n+4}{14n+3}$ is irreducible. Q.E.D.
null
null
null
By Bezout's Lemma, $3 \cdot (14n+3) - 2 \cdot (21n + 4) = 1$, so the GCD of the numerator and denominator is $1$ and the fraction is irreducible.
null
null
We notice that: So it follows that $7n+1$ and $14n+3$ must be coprime for every natural number $n$ for the fraction to be irreducible. Now the problem simplifies to proving $\frac{7n+1}{14n+3}$ irreducible. We re-write this fraction as: Since the denominator $2(7n+1) + 1$ differs from a multiple of the numerator $7n+1$ by 1, the numerator and the denominator must be relatively prime natural numbers. Hence it follows that $\frac{21n+4}{14n+3}$ is irreducible. Q.E.D
Let $\gcd (21n+4,14n+3)=a.$ So for some co-prime positive integers $x,y$ we have Multiplying $(1)$ by $2$ and $(2)$ by $3$ and then subtracting $(1)$ from $(2)$ we get We must have $a=1$, since $a$ is an positive integer. Thus, $\gcd(21n+4,14n+3)=1$ which means the fraction is irreducible, as needed.
Prove that the fraction $\frac{21n+4}{14n+3}$ is irreducible for every natural number $n$.
null
Proof by contradiction: Assume that $\dfrac{14n+3}{21n+4}$ is a reducible fraction where $p$ is the greatest common factor of $14n+3$ and $21n + 4$. Thus, Subtracting the second equation from the first equation we get $1\equiv 0\pmod{p}$ which is clearly absurd. Hence $\frac{21n+4}{14n+3}$ is irreducible. Q.E.D.
null
null
null
null
null
null
Let $N = 100a + 10b+c$ for some digits $a,b,$ and $c$. Then for some $m$. We also have $m=a^2+b^2+c^2$. Substituting this into the first equation and simplification, we get For an integer divisible by $11$, the the sum of digits in the odd positions minus the sum of digits in the even positions is divisible by $11$. Thus we get: $b = a + c$ or $b = a + c - 11$. Case $1$: Let $b=a+c$. We get Since the right side is even, the left side must also be even. Let $c=2q$ for some $q = 0,1,2,3,4$. Then Substitute $q=0,1,2,3,4$ into the last equation and then solve for $a$. When $q=0$, we get $a=5$. Thus $c=0$ and $b=5$. We get that $N=550$ which works. When $q=1$, we get that $a$ is not an integer. There is no $N$ for this case. When $q=2$, we get that $a$ is not an integer. There is no $N$ for this case. When $q=3$, we get that $a$ is not an integer. There is no $N$ for this case. When $q=4$, we get that $a$ is not an integer. There is no $N$ for this case. Case $2$: Let $b = a + c - 11$. We get Now we test all $c=0\rightarrow10$. When $c=0,1,2,4,5,6,7,8,9$, we get no integer solution to $a$. Thus, for these values of $c$, there is no valid $N$. However, when $c=3$, we get We get that $a=8$ is a valid solution. For this case, we get $a=8,b=0,c=3$, so $N=803$, and this is a valid value. Thus, the answers are $\boxed{N=550,803}$.
null
null
null
null
null
null
null
null
null
null
null
null
null
null
null
null
null
null
Note that there are only 900 three-digit numbers. Of these, only 81 are divisible by 11. This brings to mind the great strategy of listing and bashing. Being organized while listing will get you the answer. There is plenty of time on the IMO, and listing is highly unlikely to miss cases. We end up getting $\boxed{N=550,803}$.
null
null
null
null
null
null
https://artofproblemsolving.com/community/c6h54826p22231968 ~franzliszt
null
Determine all three-digit numbers $N$ having the property that $N$ is divisible by 11, and $\dfrac{N}{11}$ is equal to the sum of the squares of the digits of $N$.
null
Define a ten to be all ten positive integers which begin with a fixed tens digit. We can make a systematic approach to this: By inspection, $\dfrac{N}{11}$ must be between 10 and 90 inclusive. That gives us 8 tens to check, and 90 as well. For a given ten, the sum of the squares of the digits of $N$ increases faster than $\dfrac{N}{11}$, so we can have at most one number in every ten that works. We check the first ten: $11*11=121$ $1^2+2^2+1^2=4$ $12*11=132$ $1^2+3^2+2^2=14$ 11 is too small and 12 is too large, so all numbers below 11 will be too small and all numbers above 12 will be too large, so no numbers in the first ten work. We try the second ten: $21*11=231$ $2^2+3^2+1^2=14$ $22*11=242$ $2^2+4^2+2^2=24$ Therefore, no numbers in the second ten work. We continue, to find out that 50 and 73 are the only ones that works. $N=50*11=550$, $N=73*11=803$ so there are two $N$ that works.
null
null
null
null
null
null
null
null
null
null
null
null
null
null
null
null
null
null
null
null
null
null
null
null
null
null
null
null
null
null
null
null
null
null
(Hungary) Solve the system of equations: where $a$ and $b$ are constants. Give the conditions that $a$ and $b$ must satisfy so that $x, y, z$ (the solutions of the system) are distinct positive numbers.
Note that $x^2 + y^2 = (x+y)^2 - 2xy = (x+y)^2 - 2z^2$, so the first two equations become We note that $(x+y)^2 - z^2 = \Big[ (x+y)+z \Big]\Big[ (x+y)-z\Big]$, so if $a$ equals 0, then $b$ must also equal 0. We then have $x+y = -z$; $xy = (x+y)^2$. This gives us $x^2 + xy + y^2 = 0$. Mutiplying both sides by $(x-y)$, we have $x^3 - y^3 = 0$. Since we want $x,y$ to be real, this implies $x = y$. But $x^2 + x^2 + x^2$ can only equal 0 when $x=0$ (which, in this case, implies $y,z = 0$). Hence there are no positive solutions when $a = 0$. When $a \neq 0$, we divide $(**)$ by $(*)$ to obtain the system of equations which clearly has solution $x+y = \frac{a^2 + b^2}{2a}$, $z = \frac{a^2 - b^2}{2a}$. In order for these both to be positive, we must have positive $a$ and $a^2 > b^2$. Now, we have $x+y = \frac{a^2 + b^2}{2a}$; $xy = \left(\frac{a^2 - b^2}{2a}\right)^2$, so $x,y$ are the roots of the quadratic $m^2 - \frac{a^2 + b^2}{2a}m + \left(\frac{a^2 - b^2}{2a}\right)^2$. The discriminant for this equation is If the expressions $(3a^2 - b^2), (3b^2 - a^2)$ were simultaneously negative, then their sum, $2(a^2 + b^2)$, would also be negative, which cannot be. Therefore our quadratic's discriminant is positive when $3a^2 > b^2$ and $3b^2 > a^2$. But we have already replaced the first inequality with the sharper bound $a^2 > b^2$. It is clear that both roots of the quadratic must be positive if the discriminant is positive (we can see this either from $\left(\frac{a^2 + b^2}{2a}\right)^2 > \left(\frac{a^2 + b^2}{2a}\right)^2 - \left(2\frac{a^2 -b^2}{2a}\right)^2$ or from Descartes' Rule of Signs). We have now found the solutions to the system, and determined that it has positive solutions if and only if $a$ is positive and $3b^2 > a^2 > b^2$. Q.E.D.
null
null
null
null
null
null
null
As the new number starts with a $6$ and the old number is $1/4$ of the new number, the old number must start with a $1$. As the new number now starts with $61$, the old number must start with $\lfloor 61/4\rfloor = 15$. We continue in this way until the process terminates with the new number $615\,384$ and the old number $n=\boxed{153\,846}$.
null
null
null
null
null
null
null
null
null
null
null
null
null
null
null
null
null
null
null
null
null
null
null
null
null
null
null
Find the smallest natural number $n$ which has the following properties: (a) Its decimal representation has 6 as the last digit. (b) If the last digit 6 is erased and placed in front of the remaining digits, the resulting number is four times as large as the original number $n$.
null
Let the original number = $10n + 6$, where $n$ is a 5 digit number. Then we have $4(10n + 6) = 600000 + n$. => $40n + 24 = 600000 + n$. => $39n = 599976$. => $n = 15384$. => The original number = $\boxed{153\,846}$.
null
null
null
null
null
null
null
null
null
null
null
null
null
null
null
null
null
null
null
null
null
null
null
null
null
null
null
null
null
null
null
null
null
null
Find all real roots of the equation where $p$ is a real parameter.
Assuming $x \geq 0$, square the equation, obtaining . If we have $p + 4 \geq 4x^2$, we can square again, obtaining We must have $4 - 2p > 0 \iff p < 2$, so we have However, this is only a solution when so we have $p\geq 0$ and $p \leq \frac {4}{3}$ and $x = \frac {4 - p}{2\sqrt {4 - 2p}}$
null
null
null
null
null
null
null
We claim $2^n$ is equivalent to $2, 4,$ and $1$ $\pmod{7}$ for $n$ congruent to $1$, $2$, and $0$ $\pmod{3}$, respectively. (a) From the statement above, only $n$ divisible by $3$ will work. (b) Again from the statement above, $2^n$ can never be congruent to $-1$ $\pmod{7}$, so there are no solutions for $n$.
null
null
null
null
null
null
null
null
null
null
null
null
null
null
null
null
null
null
null
null
null
null
null
null
null
null
null
(a) Find all positive integers $n$ for which $2^n-1$ is divisible by $7$. (b) Prove that there is no positive integer $n$ for which $2^n+1$ is divisible by $7$.
null
null
null
null
null
null
null
null
null
null
null
null
null
null
null
null
null
null
null
null
null
null
null
null
null
null
null
null
null
null
null
null
null
null
null
null
Determine all values $x$ in the interval $0\leq x\leq 2\pi$ which satisfy the inequality
We shall deal with the left side of the inequality first ($2\cos x \leq \left| \sqrt{1+\sin 2x} - \sqrt{1-\sin 2x } \right|$) and the right side after that. It is clear that the left inequality is true when $\cos x$ is non-positive, and that is when $x$ is in the interval $[\pi/2, 3\pi/2]$. We shall now consider when $\cos x$ is positive. We can square the given inequality, and the resulting inequality will be true whenever the original left inequality is true. $4\cos^2{x}\leq 1+\sin 2x+1-\sin 2x-2\sqrt{1-\sin^2 2x}=2-2\sqrt{\cos^2{2x}}$. This inequality is equivalent to $2\cos^2 x\leq 1-\left| \cos 2x\right|$. I shall now divide this problem into cases. Case 1: $\cos 2x$ is non-negative. This means that $x$ is in one of the intervals $[0,\pi/4]$ or $[7\pi/4, 2\pi]$. We must find all $x$ in these two intervals such that $2\cos^2 x\leq 1-\cos 2x$. This inequality is equivalent to $2\cos^2 x\leq 2\sin^2 x$, which is only true when $x=\pi/4$ or $7\pi/4$. Case 2: $\cos 2x$ is negative. This means that $x$ is in one of the interavals $(\pi/4, \pi/2)$ or $(3\pi/2, 7\pi/4)$. We must find all $x$ in these two intervals such that $2\cos^2 x\leq 1+\cos 2x$, which is equivalent to $2\cos^2 x\leq 2\cos^2 x$, which is true for all $x$ in these intervals. Therefore the left inequality is true when $x$ is in the union of the intervals $[\pi/4, \pi/2)$, $(3\pi/2, 7\pi/4]$, and $[\pi/2, 3\pi/2]$, which is the interval $[\pi/4, 7\pi/4]$. We shall now deal with the right inequality. As above, we can square it and have it be true whenever the original right inequality is true, so we do that. $2-2\sqrt{\cos^2{2x}}\leq 2$, which is always true. Therefore the original right inequality is always satisfied, and all $x$ such that the original inequality is satisfied are in the interval $[\pi/4, 7\pi/4]$.
null
null
null
null
null
null
null
In triangle $ABC$, let $BC=a$, $AC=b$, $AB=c$, $\angle ABC=\alpha$, and $\angle BAC=2\alpha$. Using the Law of Sines gives that Therefore $\cos{\alpha}=\frac{a}{2b}$. Using the Law of Cosines gives that This can be simplified to $a^2c=b(a^2+c^2-b^2)$. Since $a$, $b$, and $c$ are positive integers, $b|a^2c$. Note that if $b$ is between $a$ and $c$, then $b$ is relatively prime to $a$ and $c$, and $b$ cannot possibly divide $a^2c$. Therefore $b$ is either the least of the three consecutive integers or the greatest. Assume that $b$ is the least of the three consecutive integers. Then either $b|b+2$ or $b|(b+2)^2$, depending on if $a=b+2$ or $c=b+2$. If $b|b+2$, then $b$ is 1 or 2. $b$ couldn't be 1, for if it was then the triangle would be degenerate. If $b$ is 2, then $b(a^2+c^2-b^2)=42=a^2c$, but $a$ and $c$ must be 3 and 4 in some order, which means that this triangle doesn't exist. therefore $b$ cannot divide $b+2$, and so $b$ must divide $(b+2)^2$. If $b|(b+2)^2$ then $b|(b+2)^2-b^2-4b=4$, so $b$ is 1, 2, or 4. Clearly $b$ cannot be 1 or 2, so $b$ must be 4. Therefore $b(a^2+c^2-b^2)=180=a^2c$. This shows that $a=6$ and $c=5$, and the triangle has sides that measure 4, 5, and 6. Now assume that $b$ is the greatest of the three consecutive integers. Then either $b|b-2$ or $b|(b-2)^2$, depending on if $a=b-2$ or $c=b-2$. $b|b-2$ is absurd, so $b|(b-2)^2$, and $b|(b-2)^2-b^2+4b=4$. Therefore $b$ is 1, 2, or 4. However, all of these cases are either degenerate or have been previously ruled out, so $b$ cannot be the greatest of the three consecutive integers. This shows that there is exactly one triangle with this property - and it has side lengths of 4, 5, and 6. $\blacksquare$ Alternate solutions are always welcome. If you have a different, elegant solution to this problem, please add it to this page.
null
null
null
null
null
null
null
null
null
null
null
null
null
null
null
null
null
null
NO TRIGONOMETRY!!! Let $a, b, c$ be the side lengths of a triangle in which $\angle C = 2\angle B.$ Extend $AC$ to $D$ such that $CD = BC = a.$ Then $\angle CDB = \frac{\angle ACB}{2} = \angle ABC$, so $ABC$ and $ADB$ are similar by AA Similarity. Hence, $c^2 = b(a+b)$. Then proceed as in Solution 2, as only algebraic manipulations are left.
null
null
null
null
null
null
null
null
Prove that there is one and only one triangle whose side lengths are consecutive integers, and one of whose angles is twice as large as another.
null
[Incomplete, please edit] In a given triangle $ABC$, let $A=2B$, $\implies C=180-3B$, and $\sin C=\sin 3B$. Then Hence, $(*)$ $a^2 = b(b+c)$ (with assumptions. This needs clearing up) If $b$ is the shortest side, $(b+2)^2 = b^2 +b(b+1)$ $\implies (b-4)(b+1)=0$, $\implies b=4, c=5, a=6$, No other permutation of $a$, $b$ and $c$ in terms of size gives integral values to $(*)$ [show]. So there is only one such triangle.
null
null
null
null
null
null
null
null
null
null
null
null
null
null
null
null
null
null
null
null
null
null
null
null
null
null
null
null
null
null
null
null
null
null
Prove that there are infinitely many natural numbers $a$ with the following property: the number $z = n^4 + a$ is not prime for any natural number $n$.
Suppose that $a = 4k^4$ for some $a$. We will prove that $a$ satisfies the property outlined above. The polynomial $n^4 + 4k^4$ can be factored as follows: Both factors are positive, because if the left one is negative, then the right one would also negative, which is clearly false. It is also simple to prove that $n^2 + 2k^2 - 2nk > 1$ when $k > 1$. Thus, for all $k > 2$, $4k^4$ is a valid value of $a$, completing the proof. $\square$ ~mathboy100 Alternate solutions are always welcome. If you have a different, elegant solution to this problem, please add it to this page.
null
null
null
null
null
null
null
null
null
null
null
null
null
null
null
null
null
null
null
null
null
null
null
null
null
null
null
null
null
null
null
null
null
null
null
Let $M$ be a point on the side $AB$ of $\triangle ABC$. Let $r_1, r_2$, and $r$ be the inscribed circles of triangles $AMC, BMC$, and $ABC$. Let $q_1, q_2$, and $q$ be the radii of the exscribed circles of the same triangles that lie in the angle $ACB$. Prove that
We use the conventional triangle notations. Let $I$ be the incenter of $ABC$, and let $I_{c}$ be its excenter to side $c$. We observe that and likewise, Simplifying the quotient of these expressions, we obtain the result Thus we wish to prove that But this follows from the fact that the angles $AMC$ and $CBM$ are supplementary.
By similar triangles and the fact that both centers lie on the angle bisector of $\angle{C}$, we have $\frac{r}{q} = \frac{s-c}{s} = \frac{a + b - c}{a + b + c}$, where $s$ is the semi-perimeter of $ABC$. Let $ABC$ have sides $a, b, c$, and let $AM = c_1, MB = c_2, MC = d$. After simple computations, we see that the condition, whose equivalent form is is also equivalent to Stewart's Theorem Alternate solutions are always welcome. If you have a different, elegant solution to this problem, please add it to this page.
null
null
null
null
null
null
null
null
null
null
null
null
null
null
null
null
null
null
null
null
null
null
null
null
null
null
null
null
null
null
null
null
null
null
Prove that the following assertion is true for $n=3$ and $n=5$, and that it is false for every other natural number $n>2:$ If $a_1, a_2,\cdots, a_n$ are arbitrary real numbers, then $(a_1-a_2)(a_1-a_3)\cdots (a_1-a_n)+(a_2-a_1)(a_2-a_3)\cdots (a_2-a_n)+\cdots+(a_n-a_1)(a_n-a_2)\cdots (a_n-a_{n-1})\ge 0.$
Take $a_1 < 0$, and the remaining $a_i = 0$. Then $E_n = a_1(n-1) < 0$ for $n$ even, so the proposition is false for even $n$. Suppose $n \ge 7$ and odd. Take any $c > a > b$, and let $a_1 = a$, $a_2 = a_3 = a_4= b$, and $a_5 = a_6 = ... = a_n = c$. Then $E_n = (a - b)^3 (a - c)^{n-4} < 0$. So the proposition is false for odd $n \ge 7$. Assume $a_1 \ge a_2 \ge a_3$. Then in $E_3$ the sum of the first two terms is non-negative, because $a_1 - a_3 \ge a_2 - a3$. The last term is also non-negative. Hence $E_3 \ge 0$, and the proposition is true for $n = 3$. It remains to prove $S_5$. Suppose $a_1 \ge a_2 \ge a_3 \ge a_4 \ge a_5$. Then the sum of the first two terms in $E_5$ is $(a_1 - a_2){(a_1 - a_3)(a_1 - a_4)(a_1 - a_5) - (a_2 - a_3)(a_2 - a_4)(a_2 - a_5)} \ge 0$. The third term is non-negative (the first two factors are non-positive and the last two non-negative). The sum of the last two terms is: $(a_4 - a_5){(a_1 - a_5)(a_2 - a_5)(a_3 - a_5) - (a_1 - a_4)(a_2 - a_4)(a_3 - a_4)} \ge 0$. Hence $E_5 \ge 0$. This solution was posted and copyrighted by e.lopes. The original thread can be fond here: [1]
null
null
null
null
null
null
null
null
null
null
null
null
null
null
null
null
null
null
null
null
null
null
null
null
null
null
null
null
null
null
null
null
null
null
null
Prove that from a set of ten distinct two-digit numbers (in the decimal system), it is possible to select two disjoint subsets whose members have the same sum.
There are $2^{10}-2=1022$ distinct subsets of our set of 10 two-digit numbers. The sum of the elements of any subset of our set of 10 two-digit numbers must be between $10$ and $90+91+92+93+94+95+96+97+98+99 < 10 \cdot 100 = 1000$. (There are fewer attainable sums.) As $1000 < 1022$, the Pigeonhole Principle implies there are two distinct subsets whose members have the same sum. Let these sets be $A$ and $B$. Now, let $A' = A - (A \cap B)$ and $B' = B - (A \cap B)$. Notice $A'$ and $B'$ are disjoint. They are also nonempty because if $A = A \cap B$ or $B = A \cap B$, then one of $A$ and $B$ is a subset of the other, so they are either not distinct or have different sums. Therefore $A'$ and $B'$ are disjoint subsets our set of 10 distinct two-digit numbers, which proves the claim. $\square$
null
null
null
null
null
null
null
null
null
null
null
null
null
null
null
null
null
null
null
null
null
null
null
null
null
null
null
null
null
null
null
null
null
null
null
Point $O$ lies on line $g;$ $\overrightarrow{OP_1}, \overrightarrow{OP_2},\cdots, \overrightarrow{OP_n}$ are unit vectors such that points $P_1, P_2, \cdots, P_n$ all lie in a plane containing $g$ and on one side of $g.$ Prove that if $n$ is odd, Here $\left|\overrightarrow{OM}\right|$ denotes the length of vector $\overrightarrow{OM}.$
We prove it by induction on the number $2n+1$ of vectors. The base step (when we have one vector) is clear, and for the induction step we use the hypothesis for the $2n-1$ vectors obtained by disregarding the outermost two vectors. We thus get a vector with norm $\ge 1$ betwen two with norm $1$. The sum of the two vectors of norm $1$ makes an angle of $\le\frac\pi 2$ with the vector of norm $\ge 1$, so their sum has norm $\ge 1$, and we're done. The above solution was posted and copyrighted by grobber. The original thread for this problem can be found here: [1] Alternate solutions are always welcome. If you have a different, elegant solution to this problem, please add it to this page.
null
null
null
null
null
null
null
null
null
null
null
null
null
null
null
null
null
null
null
null
null
null
null
null
null
null
null
null
null
null
null
null
null
null
null
Three players $A, B$ and $C$ play the following game: On each of three cards an integer is written. These three numbers $p, q, r$ satisfy $0 < p < q < r$. The three cards are shuffled and one is dealt to each player. Each then receives the number of counters indicated by the card he holds. Then the cards are shuffled again; the counters remain with the players. This process (shuffling, dealing, giving out counters) takes place for at least two rounds. After the last round, $A$ has 20 counters in all, $B$ has 10 and $C$ has 9. At the last round $B$ received $r$ counters. Who received $q$ counters on the first round?
Answer: player $C$. Let $n$ be the number of rounds played, then obviously $n (p + q + r) = 20 + 10 + 9 = 39$. So $n$ must be a divisor of 39, i. e. $n \in \{ 1, 3, 13, 39 \}$. But $p \geq 1,$ $q \geq p + 1 \geq 2,$ $r \geq q + 1 \geq 3,$ so $p + q + r \geq 1 + 2 + 3 = 6$ and $n = \frac{39}{p + q + r} \leq \frac{39}{6} < 7$. Also, by condition $n \geq 2,$ so we conclude to $n = 3$ and $p + q + r = \frac{39}{n} = 13$. As $B$ received 10 counters including $r$ counters at the last round, $10 \geq p + p + r \geq 2 + r,$ so $r \leq 8$. On the other hand, from number of counters received by $A$ we get $r > \frac{20}{3} > 6$. So $r \in \{ 7, 8 \}$. If $r = 7,$ from number of counters received by $B$ we get $3 = 10 - r \in \{ 2p, p + q, 2q \},$ and since 3 is odd, we get $p + q = 3$. But then $p + q + r = 3 + 7 = 10 \neq 13$ - a contradiction. So $r = 8$ and $2 = 10 - r \in \{ 2p, p + q, 2q \}$. On the other hand, $2 \leq 2p < p + q < 2q,$ so $2 = 2p,$ $p = 1$ and $q = 13 - p - r = 4$. It is easy to check that the only way to distribute the counters for players is $20 = r + r + q,$ $10 = p + p + r$ and $9 = q + q + p,$ so player $C$ received $q$ counters on the first round.
null
null
null
null
null
null
null
null
null
null
null
null
null
null
null
null
null
null
null
null
null
null
null
null
null
null
null
null
null
null
null
null
null
null
null
Let $x_i, y_i$ $(i=1,2,\cdots,n)$ be real numbers such that Prove that, if $z_1, z_2,\cdots, z_n$ is any permutation of $y_1, y_2, \cdots, y_n,$ then
We can expand and simplify the inequality a bit, and using the fact that $z$ is a permutation of $y$, we can cancel some terms. Consider the pairing $x_1 \rightarrow y_1$, $x_2 \rightarrow y_2$, ... $x_n \rightarrow y_n$. By switching around some of the $y$ values, we have obtained the pairing $x_1 \rightarrow z_1$, $x_2 \rightarrow z_2$, ... $x_n \rightarrow z_n$. Suppose that we switch around two $y$-values, $y_m$ and $y_n$, such that $y_m > y_n$. If $x_m > x_n$, call this a type 1 move. Otherwise, call this a type 2 move. Type 2 moves only increase the sum of the products of the pairs. The sum is increased by $x_n \cdot y_m + x_m \cdot y_n - x_m \cdot y_m - x_n \cdot y_n$. This is equivalent to $(x_n - x_m)(y_m - y_n)$, which is clearly nonnegative since $y_m > y_n$ and $x_n > x_m$. We will now consider switching from the $x$-$z$ pairing back to the $x$-$y$ pairing. We will prove that from any pairing of $x$ and $z$ values, you can use just type 2 moves to navigate back to the pairing of $x$ and $y$ values, which will complete the proof. Suppose that $x_a$ is the biggest $x$-value that is not paired with its $y$-value in the $x$ and $z$ pairing. Then, switch this $y$-value with the $y$-value currently paired with $x_a$. This is obviously a type 2 move. Continue this process until you reach back to the $x$-$y$ pairing. All moves are type 2 moves, so the proof is complete. ~mathboy100
We can rewrite the summation as Since $\sum^n_{i=1} y_i^2 = \sum^n_{i=1} z_i^2$, the above inequality is equivalent to We will now prove that the left-hand side of the inequality is the greatest sum reached out of all possible values of $\sum^n_{i=1}x_iz_i$. Obviously, if $x_1 = x_2 = \ldots = x_n$ or $y_1 = y_2 = \ldots = y_n$, the inequality is true. Now, assume, for contradiction, that neither of those conditions are true and that there exists some order of $z_i$s that are not ordered in the form $z_1 \ge z_2 \ge \ldots \ge z_n$ such that $\sum^n_{i=1}x_iz_i$ is at a maximum out of all possible permutations and is greater than the sum $\sum^n_{i=1}x_iy_i$. This necessarily means that in the sum $\sum^n_{i=1}x_iz_i$ there exists two terms $x_pz_m$ and $x_qz_n$ such that $x_p > x_q$ and $z_m < z_n$. Notice that which means if we make the terms $x_pz_n$ and $x_qz_m$ instead of the original $x_pz_m$ and $x_qz_n$, we can achieve a higher sum. However, this is impossible, since we assumed we had the highest sum. Thus, the inequality is proved, which is equivalent to what we wanted to prove. ~Imajinary
null
null
null
null
null
null
null
null
null
null
null
null
null
null
null
null
null
null
null
null
null
null
null
null
null
null
null
null
null
null
null
null
null
null
In a convex quadrilateral (in the plane) with the area of $32 \text{ cm}^{2}$ the sum of two opposite sides and a diagonal is $16 \text{ cm}$. Determine all the possible values that the other diagonal can have.
Label the vertices $A$, $B$, $C$, and $D$ in such a way that $AB + BD + DC = 16$, and $\overline{BD}$ is a diagonal. The area of the quadrilateral can be expressed as $BD \cdot ( d_1 + d_2 ) / 2$, where $d_1$ and $d_2$ are altitudes from points $A$ and $C$ onto $\overline{BD}$. Clearly, $d_1 \leq AB$ and $d_2 \leq DC$. Hence the area is at most $BD \cdot ( AB + DC ) / 2 = BD(16-BD) / 2$. The quadratic function $f(x)=x(16-x)/2$ has its maximum for $x=8$, and its value is $f(8)=32$. The area of our quadrilateral is $32$. This means that we must have $BD=8$. Also, equality must hold in both $d_1 \leq AB$ and $d_2 \leq DC$. Hence both $\overline{AB}$ and $\overline{DC}$ must be perpendicular to $\overline{BD}$. And in any such case it is clear from the Pythagorean theorem that $AC = 8\sqrt 2$. Therefore the other diagonal has only one possible length: $8\sqrt 2$.
null
null
null
null
null
null
null
null
null
null
null
null
null
null
null
null
null
null
null
null
null
null
null
null
null
null
null
null
null
null
null
null
null
null
null
In the interior of a square $ABCD$ we construct the equilateral triangles $ABK, BCL, CDM, DAN.$ Prove that the midpoints of the four segments $KL, LM, MN, NK$ and the midpoints of the eight segments $AK, BK, BL, CL, CM, DM, DN, AN$ are the 12 vertices of a regular dodecagon.
Just use complex numbers, with $a = 1$, $b = i$, $c = - 1$ and $d = - i$. With some calculations, we have $k = \frac {\sqrt {3} - 1}{2}( - 1 - i)$, $l = \frac {\sqrt {3} - 1}{2}(1 - i)$, $m = \frac {\sqrt {3} - 1}{2}(1 + i)$ and $n = \frac {\sqrt {3} - 1}{2}( - 1 + i)$. Now it's an easy job to calculate the twelve midpoints and to find out they are all of the form $\frac {\sqrt {3} - 1}{2}e^{\frac {k\pi}{6}i}$, with $k\in\mathbb{N}: 0\le k\le 11$, and the result follows. The above solution was posted and copyrighted by Joao Pedro Santos. The original thread for this problem can be found here: [1]
null
null
null
null
null
null
null
null
null
null
null
null
null
null
null
null
null
null
null
null
null
null
null
null
null
null
null
null
null
null
null
null
null
null
null
Let $m$ and $n$ be positive integers such that $1 \le m < n$. In their decimal representations, the last three digits of $1978^m$ are equal, respectively, to the last three digits of $1978^n$. Find $m$ and $n$ such that $m + n$ has its least value.
We have $1978^m\equiv 1978^n\pmod {1000}$, or $978^m-978^n=1000k$ for some positive integer $k$ (if it is not positive just do $978^n-978^m=-1000k$). Hence $978^n\mid 1000k$. So dividing through by $978^n$ we get $978^{m-n}-1=\frac{1000k}{978^n}$. Observe that $2\nmid LHS$, so $2\nmid RHS$. So since $2|| 978^n$, clearly the minimum possible value of $n$ is $3$ (and then $489^n\mid k$). We will show later that if $n$ is minimal then $m$ is minimal. We have $978^{m-3}-1\equiv 0\pmod {125}\Leftrightarrow 103^{m-3}\equiv 1\pmod {125}$. Hence, $m-3\mid \varphi(125)\Rightarrow m-3\mid 100$. Checking by hand we find that only $m-3=100$ works (this also shows that minimality of $m$ depends on $n$, as claimed above). So $m=103$. Consequently, $m+n=106$ with $\boxed{(m,n)=(103,3)}$. The above solution was posted and copyrighted by cobbler and Andreas. The original thread for this problem can be found here: [1] and [2]
null
null
null
null
null
null
null
null
null
null
null
null
null
null
null
null
null
null
null
null
null
null
null
null
null
null
null
null
null
null
null
null
null
null
null
If $p$ and $q$ are natural numbers so thatprove that $p$ is divisible with $1979$.
We first write Now, observe that and similarly $\frac{1}{661}+\frac{1}{1318}=\frac{1979}{661\cdot 1318}$ and $\frac{1}{662}+\frac{1}{1317}=\frac{1979}{662\cdot 1317}$, and so on. We see that the original equation becomes where $s=660\cdot 661\cdots 1319$ and $r=\frac{s}{660\cdot 1319}+\frac{s}{661\cdot 1318}+\cdots+\frac{s}{989\cdot 990}$ are two integers. Finally consider $p=1979\cdot\frac{qr}{s}$, and observe that $s\nmid 1979$ because $1979$ is a prime, it follows that $\frac{qr}{s}\in\mathbb{Z}$. Hence we deduce that $p$ is divisible with $1979$. The above solution was posted and copyrighted by Solumilkyu. The original thread for this problem can be found here: [1]
null
null
null
null
null
null
null
null
null
null
null
null
null
null
null
null
null
null
null
null
null
null
null
null
null
null
null
null
null
null
null
null
null
null
null
$P$ is a point inside a given triangle $ABC$. $D, E, F$ are the feet of the perpendiculars from $P$ to the lines $BC, CA, AB$, respectively. Find all $P$ for which is least.
We note that $BC \cdot PD + CA \cdot PE + AB \cdot PF$ is twice the triangle's area, i.e., constant. By the Cauchy-Schwarz Inequality, with equality exactly when $PD = PE = PF$, which occurs when $P$ is the triangle's incenter, Q.E.D. Alternate solutions are always welcome. If you have a different, elegant solution to this problem, please add it to this page.
null
null
null
null
null
null
null
Clearly $f(1) \ge 1 \Rightarrow f(m+1) \ge f(m)+f(1) \ge f(m)+1$ so $f(9999) \ge 9999$.Contradiction!So $f(1)=0$.This forces $f(3)=1$.Hence $f(3k+3) \ge f(3k)+f(3)>f(3k)$ so the inequality $f(3)<f(6)<\cdots<f(9999)=3333$ forces $f(3k)=k \forall k \le 3333$.Now $f(3k+2) \ge k+1 \Rightarrow f(6k+4) \ge 2k+2 \Rightarrow f(12k+8) \ge 4k+4 \le f(12k+9)=4k+3$(Note:This is valid for $12k+9 \le 9999$ or $3k+2 \le 2499$).Contradiction!Hence the non-decreasing nature of $f$ gives $f(3k+1)=k$.Hence $f(n)=\lfloor\frac{n}{3}\rfloor \forall 1\le n \le 2499$. So $f(1982)=\lfloor\frac{1982}{3}\rfloor=660$. This solution was posted and copyrighted by sayantanchakraborty. The original thread for this problem can be found here: [1]
null
null
null
null
null
null
null
null
null
null
null
null
null
null
null
null
null
null
We show that $f(n) = [n/3]$ for $n <= 9999$, where [ ] denotes the integral part. We show first that $f(3) = 1$. $f(1)$ must be $0$, otherwise $f(2) - f(1) - f(1)$ would be negative. Hence $f(3) = f(2) + f(1) + 0$ or $1$ = $0$ or $1$. But we are told $f(3) > 0$, so $f(3) = 1$. It follows by induction that $f(3n) >= n$. For $f(3n+3) = f(3) + f(3n)$ + $0$ or $1 = f(3n) + 1$ or $2$. Moreover if we ever get $f(3n) > n$, then the same argument shows that $f(3m) > m$ for all $m > n$. But $f(3.3333) = 3333$, so $f(3n) = n$ for all $n <= 3333$. Now $f(3n+1) = f(3n) + f(1) + 0$ or $1$ = $n$ or $n + 1$. But $3n+1 = f(9n+3) >= f(6n+2) + f(3n+1) >= 3f(3n+1)$, so $f(3n+1) < n+1$. Hence $f(3n+1) = n.$ Similarly, $4f(3n+2) = n$. In particular $f(1982) = 660$. This solution was posted and copyrighted by Tega. The original thread for this problem can be found here: [3]
null
null
null
null
null
null
Similar to solution 3. Proof: Lemma 1: $f(mn)\geq nf(m)$ Let, $P(m,m)$ be assertion. Similarly,we can induct to get $f(nm)\geq nf(m)$. Lemma proved. Then we see that, Then, Then we can easily get,by assertion $P(1980,2)$ Hence, $\boxed{f(1980)=660,661}$.And, we are done. $\blacksquare$ This solution was posted and copyrighted by IMO2019. The original thread for this problem can be found here: [4]
null
The function $f(n)$ is defined on the positive integers and takes non-negative integer values. $f(2)=0,f(3)>0,f(9999)=3333$ and for all $m,n:$Determine $f(1982)$.
null
First observe that Since $f(3)$ is a positive integer, we need $f(3)=1$. Next, observe that \begin{align*} 3333=f(9999)\geq 5f(1980)+33f(3)=5f(1980)+33\quad\Longrightarrow\quad f(1980)\leq 660 \end{align*}On the other hand, $f(1980)\geq 660f(3)=660$, so combine the two inequalities we obtain $f(1980)=660$. Finally, write Suppose that $f(1982)=661$, then \begin{align*} 3333=f(9999)\geq 5f(1982)+29f(3)=3305+29=3334 \end{align*}a contradiction. Hence we conclude that $f(1982)=660$. This solution was posted and copyrighted by Solumilkyu. The original thread for this problem can be found here: [2]
null
null
null
null
null
null
Let $x=y=1$ and we have $f(f(1))=f(1)$. Now, let $x=1,y=f(1)$ and we have $f(f(f(1)))=f(1)f(1)\Rightarrow f(1)=[f(1)]^2$ since $f(1)>0$ we have $f(1)=1$. Plug in $y=x$ and we have $f(xf(x))=xf(x)$. If $a=1$ is the only solution to $f(a)=a$ then we have $xf(x)=1\Rightarrow f(x)=\frac{1}{x}$. We prove that this is the only function by showing that there does not exist any other $a$: Suppose there did exist such an $a\ne1$. Then, letting $y=a$ in the functional equation yields $f(xa)=af(x)$. Then, letting $x=\frac{1}{a}$ yields $f(\frac{1}{a})=\frac{1}{a}$. Notice that since $a\ne1$, one of $a,\frac{1}{a}$ is greater than $1$. Let $b$ equal the one that is greater than $1$. Then, we find similarly (since $f(b)=b$) that $f(xb)=bf(x)$. Putting $x=b$ into the equation, yields $f(b^2)=b^2$. Repeating this process we find that $f(b^{2^k})=b^{2^k}$ for all natural $k$. But, since $b>1$, as $k\to \infty$, we have that $b^{2^k}\to\infty$ which contradicts the fact that $f(x)\to0$ as $x\to \infty$.
null
null
null
null
null
null
null
null
null
null
null
null
null
null
null
null
null
null
null
null
null
null
null
null
null
null
null
Find all functions $f$ defined on the set of positive reals which take positive real values and satisfy the conditions: (i) $f(xf(y))=yf(x)$ for all $x,y$; (ii) $f(x)\to0$ as $x\to \infty$.
null
Let $x=1$ so If $f(a)=f(b),$ then $af(1)=f(f(a))=f(f(b))=bf(1)\implies a=b$ because $f(1)$ goes to the real positive integers, not $0.$ Hence, $f$ is injective. Let $x=y$ so so $xf(x)$ is a fixed point of $f.$ Then, let $y=1$ so $f(xf(1))=f(x)\implies f(1)=1$ as $x$ can't be $0$ so $1$ is a fixed point of $f.$ We claim $1$ is the only fixed point of $f.$ Suppose for the sake of contradiction that $a,b$ be fixed points of $f$ so $f(a)=a$ and $f(b)=b.$ Then, setting $x=a,y=b$ in (i) gives so $ab$ is also a fixed point of $f.$ Also, let $x=\frac{1}{a},y=a$ so so $\frac{1}{a}$ is a fixed point of $f.$ If $f(a)=a$ with $a>1,$ then $f(a^n)$ is a fixed point of $f$, contradicting (ii). If $f(a)=a$ with $0<a<1,$ then $f(\frac{1}{a^n})=\frac{1}{a^n}$ so $\frac{1}{a^n}$ is a fixed point, contradicting (ii). Hence, the only fixed point is $1$ so $xf(x)=1$ so $f(x)=\boxed{\frac{1}{x}}$ and we can easily check that this solution works.
null
null
null
null
null
null
null
null
null
null
null
null
null
null
null
null
null
null
null
null
null
null
null
null
null
null
null
null
null
null
null
null
null
null
Let $x$, $y$, $z$ be nonnegative real numbers with $x + y + z = 1$. Show that $0 \leq xy+yz+zx-2xyz \leq \frac{7}{27}$
Note that this inequality is symmetric with x,y and z. To prove note that $x+y+z=1$ implies that at most one of $x$, $y$, or $z$ is greater than $\frac{1}{2}$. Suppose $x \leq \frac{1}{2}$, WLOG. Then, $xy+yz+zx-2xyz=yz(1-2x)+xy+zx\geq 0$ since $(1-2x)\geq 0$, implying all terms are positive. To prove $xy+yz+zx-2xyz \leq \frac{7}{27}$, suppose $x \leq y \leq z$. Note that $x \leq y \leq \frac{1}{2}$ since at most one of x,y,z is $\frac{1}{2}$. Suppose not all of them equals $\frac{1}{3}$-otherwise, we would be done. This implies $x \leq \frac{1}{3}$ and $z \geq \frac{1}{3}$. Thus, define , Then, $x'=x+\epsilon$, $z'=z-\epsilon$, and $x'+y'+z'=1$. After some simplification, since $1-2y>0$ and $z-x-\epsilon=z-\frac{1}{3}>0$. If we repeat the process, defining after similar reasoning, we see that .
null
null
null
null
null
Lemma. Let $I$ be the in-center of $ABC$ and points $P$ and $Q$ be on the lines $AB$ and $BC$ respectively. Then $BP + CQ = BC$ if and only if $APIQ$ is a cyclic quadrilateral. Solution. Assume that rays $AD$ and $BC$ intersect at point $P$. Let $S$ be the center od circle touching $AD$, $DC$ and $CB$. Obviosuly $S$ is a $P$-ex-center of $PDB$, hence $\angle DSI=\angle DSP = \frac{1}{2} \angle DCP=\frac{1}{2} \angle A=\angle DAI$ so DASI is concyclic.
null
Let $O$ be the center of the circle mentioned in the problem. Let $T$ be the second intersection of the circumcircle of $CDO$ with $AB$. By measures of arcs, $\angle DTA = \angle DCO = \frac{\angle DCB}{2} = \frac{\pi}{2} - \frac{\angle DAB}{2}$. It follows that $AT = AD$. Likewise, $TB = BC$, so $AD + BC = AB$, as desired.
null
null
null
null
null
null
null
null
null
null
null
null
null
null
null
null
null
null
Let the circle have center $O$ and radius $r$, and let its points of tangency with $BC, CD, DA$ be $E, F, G$, respectively. Since $OEFC$ is clearly a cyclic quadrilateral, the angle $COE$ is equal to half the angle $GAO$. Then Likewise, $DG = OB - EB$. It follows that Q.E.D.
null
null
null
This solution is incorrect. The fact that $BC$ is tangent to the circle does not necessitate that $B$ is its point of tangency. -Nitinjan06 From the fact that AD and BC are tangents to the circle mentioned in the problem, we have $\angle{CBA}=90\deg$ and $\angle{DAB}=90\deg$. Now, from the fact that ABCD is cyclic, we obtain that $\angle{BCD}=90\deg$ and $\angle{CDA}=90\deg$, such that ABCD is a rectangle. Now, let E be the point of tangency between the circle and CD. It follows, if O is the center of the circle, that $\angle{OEC}=\angle{OED}=90\deg$ Since $AO=EO=BO$, we obtain two squares, $AOED$ and $BOEC$. From the properties of squares we now have $AD+BC=AO+BO=AB$ as desired.
null
null
We use the notation of the previous solution. Let $X$ be the point on the ray $AD$ such that $AX = AO$. We note that $OF = OG = r$; $\angle OFC = \angle OGX = \frac{\pi}{2}$; and $\angle FCO = \angle GXO = \frac{\pi - \angle BAD}{2}$; hence the triangles $OFC, OGX$ are congruent; hence $GX = FC = CE$ and $AO = AG + GX = AG + CE$. Similarly, $OB = EB + GD$. Therefore $AO + OB = AG + GD + CE + EB$, Q.E.D.
null
A circle has center on the side $AB$ of the cyclic quadrilateral $ABCD$. The other three sides are tangent to the circle. Prove that $AD + BC = AB$.
null
Let $O$ be the center of the circle mentioned in the problem, and let $T$ be the point on $AB$ such that $AT = AD$. Then $\angle DTA = \frac{ \pi - \angle DAB}{2} = \angle DCO$, so $DCOT$ is a cyclic quadrilateral and $T$ is in fact the $T$ of the previous solution. The conclusion follows.
null
null
null
null
null
null
We do casework with modular arithmetic. $d\equiv 0,3 \pmod{4}: 13d-1$ is not a perfect square. $d\equiv 2\pmod{4}: 2d-1$ is not a perfect square. Therefore, $d\equiv 1, \pmod{4}.$ Now consider $d\pmod{16}.$ $d\equiv 1,13 \pmod{16}: 13d-1$ is not a perfect square. $d\equiv 5,9\pmod{16}: 5d-1$ is not a perfect square. As we have covered all possible cases, we are done.
null
null
null
null
null
null
null
null
null
null
null
null
null
null
null
null
null
null
null
null
null
null
null
null
null
null
null
Let $d$ be any positive integer not equal to $2, 5$ or $13$. Show that one can find distinct $a,b$ in the set $\{2,5,13,d\}$ such that $ab-1$ is not a perfect square.
null
Proof by contradiction: Suppose $p^2=2d-1$, $q^2=5d-1$ and $r^2=13d-1$. From the first equation, $p$ is an odd integer. Let $p=2k-1$. We have $d=2k^2-2k+1$, which is an odd integer. Then $q^2$ and $r^2$ must be even integers, denoted by $4n^2$ and $4m^2$ respectively, and thus $r^2-q^2=4m^2-4n^2=8d$, from which can be deduced. Since $m^2-n^2$ is even, $m$ and $n$ have the same parity, so $(m+n)(m-n)$ is divisible by $4$. It follows that the odd integer $d$ must be divisible by $2$, leading to a contradiction. We are done. Alternate solutions are always welcome. If you have a different, elegant solution to this problem, please add it to this page.
null
null
null
null
null
null
null
null
null
null
null
null
null
null
null
null
null
null
null
null
null
null
null
null
null
null
null
null
null
null
null
null
null
null
Let $p_n (k)$ be the number of permutations of the set $\{ 1, \ldots , n \} , \; n \ge 1$, which have exactly $k$ fixed points. Prove that (Remark: A permutation $f$ of a set $S$ is a one-to-one mapping of $S$ onto itself. An element $i$ in $S$ is called a fixed point of the permutation $f$ if $f(i) = i$.)
The sum in question simply counts the total number of fixed points in all permutations of the set. But for any element $i$ of the set, there are $(n-1)!$ permutations which have $i$ as a fixed point. Therefore as desired. Slightly Clearer Solution For any $k$, if there are $p_n(k)$ permutations that have $k$ fixed points, then we know that each fixed point is counted once in the product $k \cdot p_n{k}$. Therefore the given sum is simply the number of fixed points among all permutations of $\{ 1, \ldots , n \}$. However, if we take any $x$ such that $1 \le x \le n$ and $x$ is a fixed point, there are $(n-1)!$ ways to arrange the other numbers in the set. Therefore our desired sum becomes $n \cdot (n-1)! = n!$, so we are done.
The probability of any number $i$ where $1\le i\le n$ being a fixed point is $\frac{1}{n}$. Thus, the expected value of the number of fixed points is $n\times \frac{1}{n}=1$. The expected value is also $\sum_{k=0}^{n} \frac{k \cdot p_n (k)}{n!}$. Thus, or
null
null
null
null
null
null
null
null
null
null
null
null
null
null
null
null
null
null
null
null
null
null
null
null
null
null
null
null
null
null
null
null
null
null
Prove that in the set $\{1,2, \ldots, 1989\}$ can be expressed as the disjoint union of subsets $A_i, \{i = 1,2, \ldots, 117\}$ such that i.) each $A_i$ contains 17 elements ii.) the sum of all the elements in each $A_i$ is the same.
Let us start pairing numbers in the following fashion, where each pair sums to $1990$: $(1, 1990-1), (2, 1990-2), \ldots (936, 1054)$ There are a total of $117*8$ pairs above. Let us start putting these pairs into each of the $117$ subsets starting with the first pair $(1, 1990-1)$ going into $A_1$, $(2, 1990-2)$ into $A_2$ and so on $\ldots$ with $(k, 1990-k)$ going into $A_{k(mod 117)}$. Now we have $16$ numbers present in each of the $117$ subsets all of which have the same total sum. We need $1$ more number to be filled in each subset from the remaining $117$ numbers $(937, 938, ... 1053)$. Let us arrange these $117$ numbers in the following manner: $995-58, 995-57, \ldots, 995-1, 995, 995+1, 995+2, \ldots, 995+57, 995+58$ $==> (1)$ Now notice that for each number that's off by $x$ from $995$, there's a counter number off by $x$ from $995$ in the opposite direction. So we need to create similar imbalances in the Subsets $A_1$ to $A_{117}$, so that we could add the above $117$ numbers to those imbalances to keep the total sum unchanged. Now start swapping the $1st$ number of each pair that we added to subsets $A_1$ to $A_{117}$ to create the above imbalances: Swap $1$ from $A_1$ with $59$ from $A_{59}$ - this creates an imbalance of $+58$ and $-58$ Swap $2$ from $A_2$ with $58$ from $A_{58}$ - this creates an imbalance of $+56$ and $-56$ $\ldots$ Swap $29$ from $A_{29}$ with $31$ from $A_{31}$ - creates an imbalance of $+2$ and $-2$ Leave $30$ in $A_{30}$ as it is - $0$ imbalance. Similarly, Swap $60$ from $A_{60}$ with $117$ from $A_{117}$ - this creates an imbalance of $+57$ and $-57$ Swap $61$ from $A_{61}$ with $116$ from $A_{116}$ - this creates an imbalance of $+55$ and $-55$ $\ldots$ Swap $88$ from $A_{88}$ with $89$ from $A_{89}$ - this creates an imbalance of $+1$ and $-1$ Now start adding the $117$ numbers from $(1)$ to the subsets $A_1$ to $A_{117}$ to counter the imbalances $-58$ to $+58$ so that the sum remains unchanged. Notice that each subset now has $17$ elements with total sum = $8*1990 + 995 = 16915$. $- Kris17$
null
null
null
null
null
null
null
With simple angle chasing, we find that triangles $CEG$ and $MDB$ are similar. so, $GE/EC = MD/MB$. (*) again with simple angle chasing, we find that triangles $CEF$ and $AMD$ are similar. so, $EF/DM = CE/AM$. (**) so, by (*) and (**), we have $GE/EF = MA/MB = t/(t-1)$. This solution was posted and copyrighted by e.lopes. The original thread for this problem can be found here: [1]
null
null
null
null
null
null
null
null
null
null
null
null
null
null
null
null
null
null
null
null
null
null
null
null
null
null
null
Chords $AB$ and $CD$ of a circle intersect at a point $E$ inside the circle. Let $M$ be an interior point of the segment $\overline{EB}$. The tangent line at $E$ to the circle through $D, E$, and $M$ intersects the lines $\overline{BC}$ and ${AC}$ at $F$ and $G$, respectively. If $\frac{AM}{AB} = t$, find $\frac{EG}{EF}$ in terms of $t$.
null
This problem can be bashed with PoP and Ratio Lemma. Rewriting the given ratio gets $\frac{MA}{MB}=\frac{t}{1-t}$. By Ratio Lemma, $\frac{FB}{FC}=\frac{BE}{CE} \cdot \frac{\sin{\angle{FEB}}}{\sin{\angle{FEC}}}=\frac{DE}{AE} \cdot \frac{\sin{\angle{EDM}}}{\sin{\angle{DME}}}=\frac{DE}{AE} \cdot \frac{EM}{DE}=\frac{ME}{EA}$. Similarly, $\frac{GA}{GC}=\frac{ME}{EB}$. We can rewrite these equalities to get $\frac{AM}{EM}=\frac{BC}{FB}$ and $\frac{BM}{EM}=\frac{AC}{GC}$. Using Ratio Lemma, $\frac{GE}{\sin{\angle{ACD}}}=\frac{GC}{\sin{\angle{GED}}}$ and $\frac{EF}{\sin{\angle{BCD}}}=\frac{FC}{\sin{\angle{FEC}}}$. Since $\angle{GED}=\angle{FEC}$, we have $\frac{FE}{GE}=\frac{FC}{GC} \cdot \frac{\sin{\angle{BCD}}}{\sin{\angle{ACD}}}$ (eq 1). Note that by Ratio Lemma, $\frac{\sin{\angle{BCD}}}{\sin{\angle{ACD}}}=\frac{CA}{CB} \cdot \frac{EB}{EA}$. Plugging this into (eq 1), we get $\frac{EF}{GE}=\frac{FC}{GC} \cdot \frac{CA}{CB} \cdot \frac{EB}{EA}=\frac{\frac{EA}{EM} \cdot FB}{\frac{EB}{EM} \cdot GA} \cdot \frac{CA}{CB} \cdot \frac{EB}{EA}=\frac{FB}{GA} \cdot \frac{CA}{CB}=\frac{EM}{AM} \cdot \frac{MB}{EM}=\frac{MB}{MA}=\frac{1-t}{t}$. So $\frac{EG}{EF}=\boxed{\frac{t}{1-t}}$. This solution was posted and copyrighted by AIME12345. The original thread for this problem can be found here: [2]
null
null
null
null
null
null
null
null
null
null
null
null
null
null
null
null
null
null
null
null
null
null
null
null
null
null
null
null
null
null
null
null
null
null
Find all integers $a$, $b$, $c$ satisfying $1 < a < b < c$ such that $(a - 1)(b -1)(c - 1)$ is a divisor of $abc - 1$.
$1<\frac{abc-1}{(a-1)(b-1)(c-1)}<\frac{abc}{(a-1)(b-1)(c-1)}$ $1<\frac{abc-1}{(a-1)(b-1)(c-1)}<\left(\frac{a}{a-1}\right) \left(\frac{b}{b-1}\right) \left(\frac{c}{c-1}\right)$ With $1<a<b<c$ it implies that $a \ge 2$, $b \ge 3$, $c \ge 4$ Therefore, $\frac{a}{a-1}=1+\frac{1}{a-1}$ which for $a$ gives: $\frac{a}{a-1} \le 1+\frac{1}{2-1}$, which gives :$\frac{a}{a-1} \le 2$ for $b$ gives: $\frac{b}{b-1} \le 1+\frac{1}{3-1}$, which gives :$\frac{b}{b-1} \le \frac{3}{2}$ for $c$ gives: $\frac{c}{c-1} \le 1+\frac{1}{4-1}$, which gives :$\frac{c}{c-1} \le \frac{4}{3}$ Substituting those inequalities into the original inequality gives: $1<\frac{abc-1}{(a-1)(b-1)(c-1)}<\left( 2 \right) \left(\frac{3}{2}\right) \left(\frac{4}{3}\right)$ $1<\frac{abc-1}{(a-1)(b-1)(c-1)}<4$ Since $\frac{abc-1}{(a-1)(b-1)(c-1)}$ needs to be integer, then $\frac{abc-1}{(a-1)(b-1)(c-1)}=2$ or $\frac{abc-1}{(a-1)(b-1)(c-1)}=3$ Case 1: $\frac{abc-1}{(a-1)(b-1)(c-1)}=2$ $abc-1=2(a-1)(b-1)(c-1)=2abc-2(ab+bc+ac)+2(a+b+c)-2$ Case 1, subcase $a=2$: $2bc-1=2bc-2(b+c)+2$ gives: $2(b+c)=3$ which has no solution because $2(b+c)$ is even. Case 1, subcase $a=3$: $3bc-1=4bc-4(b+c)+4$ $bc-4b-4c+5=0$ $(b-4)(c-4)=11$ $b-4=1$ and $c-4=11$ provides solution $(a,b,c)=(3,5,15)$ Case 2: $\frac{abc-1}{(a-1)(b-1)(c-1)}=3$ $abc-1=3(a-1)(b-1)(c-1)=3abc-3(ab+bc+ac)+3(a+b+c)-3$ Case 2, subcase $a=2$: $2bc-1=3bc-3(b+c)+3$ $bc-3b-3c+4=0$ $(b-3)(c-3)=5$ $b-3=1$ and $c-3=5$ provides solution $(a,b,c)=(2,4,8)$ Case 2, subcase $a=3$: $3bc-1=6bc-6(b+c)+6$ Since $(3bc-1$) mod $3 = -1$ and $(6bc-6(b+c)+6)$ mod $3 = 0$, then there is no solution for this subcase. Now we verify our two solutions: when $(a,b,c)=(2,4,8)$ $abc-1=(2)(4)(8)-1=63$ and $(a-1)(b-1)(c-1)=(1)(3)(7)=21$ Since $21$ is a factor of $63$, this solutions is correct. when $(a,b,c)=(3,5,15)$ $abc-1=(3)(5)(15)-1=224$ and $(a-1)(b-1)(c-1)=(2)(4)(14)=112$ Since $112$ is a factor of $224$, this solutions is also correct. The solutions are: $(a,b,c)=(2,4,8)$ and $(a,b,c)=(3,5,15)$ ~ Tomas Diaz. orders@tomasdiaz.com Alternate solutions are always welcome. If you have a different, elegant solution to this problem, please add it to this page.
null
null
null
null
null
null
null
null
null
null
null
null
null
null
null
null
null
null
null
null
null
null
null
null
null
null
null
null
null
null
null
null
null
null
null
Let $f\left(x\right)=x^n+5x^{n-1}+3$, where $n>1$ is an integer. Prove that $f\left(x\right)$ cannot be expressed as the product of two non-constant polynomials with integer coefficients.
For the sake of contradiction, assume that $f\left(x\right)=g\left(x\right)h\left(x\right)$ for polynomials $g\left(x\right)$ and $h\left(x\right)$ in $\mathbb{R}$. Furthermore, let $g\left(x\right)=b_mx^m+b_{m-1}x^{m-1}+\ldots+b_1x+b_0$ with $b_i=0$ if $i>m$ and $h\left(x\right)=c_{n-m}x^{n-m}+c_{n-m-1}x^{n-m-1}+\ldots+c_1x+c_0$ with $c_i=0$ if $i>n-m$. This gives that $f\left(x\right)=\sum_{i=0}^{n}\left(\sum_{j=0}^{i}b_jc_{i-j}\right)x^i$. We have that $3=b_0c_0$, or $3|b_0c_0$. WLOG, let $3|b_0$ (and thus $3\not|c_0$). Since $b_0c_1+b_1c_0=0$ and $3$ divides $b_0$ but not $c_0$, we need that $3|b_1$. We can keep on going up the chain until we get that $3|b_{n-2}$. Then, by equating coefficients once more, we get that $b_0c_{n-1}+b_1c_{n-2}+\ldots+b_{n-2}c_1+b_{n-1}c_0=5$. Taking the equation $\pmod3$ gives that $b_{n-1}c_0\equiv2\pmod3$. This implies that $b_{n-1}\neq0$. Thus, the degree of $g\left(x\right)$ is at least $n-1$. However, because $h\left(x\right)$ is a non-constant factor of $f\left(x\right)$, we have that the degree of $g\left(x\right)$ is at most $n-1$. Thus, the degree of $g\left(x\right)$ is $n-1$, implying that the degree of $h\left(x\right)$ is $1$. From this fact, we have that there must exist a rational root of $f\left(x\right)$. The only candidates are $1$, $-1$, $3$, and $-3$, though. $f(x)\equiv x+5x+3\equiv1\pmod3$ when $x$ is an integer, so none of these work. Thus, there are no linear factors of $f\left(x\right)$. In other words, $f\left(x\right)$ cannot be expressed as $g\left(x\right)h\left(x\right)$ for polynomials $g\left(x\right)$ and $h\left(x\right)$ in $\mathbb{R}$. This means that $f\left(x\right)$ cannot be expressed as the product of two non-constant polynomials with integer coefficients. Q.E.D.
null
null
null
null
null
null
null
null
null
null
null
null
null
null
null
null
null
null
null
null
null
null
null
null
null
null
Let $AM$ and $XY$ intersect at $Z$. Because $\angle AMC = \angle BND = \angle APT = 90^\circ$, we have quadrilaterals $AMPT$ and $DNPT$ cyclic. Therefore, $Z$ lies on the radical axis of the two circumcircles of these quadrilaterals. But $Z$ also lies on radical axis $XY$ of the original two circles, so the power of $Z$ with respect to each of the four circles is all equal to $ZM * ZA$. Hence, $Z$ lies on the radical axis $DN$ of the two circles passing through $D$ and $N$, as desired. what is $T$ here?
null
null
null
null
null
null
null
null
Let $A,B,C,D$ be four distinct points on a line, in that order. The circles with diameters $AC$ and $BD$ intersect at $X$ and $Y$. The line $XY$ meets $BC$ at $Z$. Let $P$ be a point on the line $XY$ other than $Z$. The line $CP$ intersects the circle with diameter $AC$ at $C$ and $M$, and the line $BP$ intersects the circle with diameter $BD$ at $B$ and $N$. Prove that the lines $AM,DN,XY$ are concurrent.
Since $M$ is on the circle with diameter $AC$, we have $\angle AMC=90$ and so $\angle MCA=90-A$. We similarly find that $\angle BND=90$. Also, notice that the line $XY$ is the radical axis of the two circles with diameters $AC$ and $BD$. Thus, since $P$ is on $XY$, we have $PN\cdot PB=PM\cdot PC$ and so by the converse of Power of a Point, the quadrilateral $MNBC$ is cyclic. Thus, $90-A=\angle MCA=\angle BNM$. Thus, $\angle MND=180-A$ and so quadrilateral $AMND$ is cyclic. Let the name of the circle $AMND$ be $O$ . Then, the radical axis of $O$ and the circle with diameter $AC$ is line $AM$. Also, the radical axis of $O$ and the circle with diameter $BD$ is line $DN$. Since the pairwise radical axes of 3 circles are concurrent, we have $AM,DN,XY$ are concurrent as desired.
Let $AM$ and $PT$ (a subsegment of $XY$) intersect at $Z$. Now, assume that $Z, N, P$ are not collinear. In that case, let $ZD$ intersect the circle with diameter $BD$ at $N'$ and the circle through $D, P, T$ at $N''$. We know that $\angle AMC = \angle BND = \angle ATP = 90^\circ$ via standard formulae, so quadrilaterals $AMPT$ and $DNPT$ are cyclic. Thus, $N'$ and $N''$ are distinct, as none of them is $N$. Hence, by Power of a Point, However, because $Z$ lies on radical axis $TP$ of the two circles, we have Hence, $ZN'' = ZN'$, a contradiction since $D$ and $D'$ are distinct. We therefore conclude that $Z, N, D$ are collinear, which gives the concurrency of $AM, XY$, and $DN$. This completes the problem.
null
null
null
null
null
null
null
null
null
null
null
null
null
null
null
null
null
null
null
null
null
null
null
null
null
null
null
null
null
null
null
null
null
null
We are given a positive integer $r$ and a rectangular board $ABCD$ with dimensions $|AB|=20$, $|BC|=12$. The rectangle is divided into a grid of $20 \times 12$ unit squares. The following moves are permitted on the board: one can move from one square to another only if the distance between the centers of the two squares is $\sqrt{r}$. The task is to find a sequence of moves leading from the square with $A$ as a vertex to the square with $B$ as a vertex. (a) Show that the task cannot be done if $r$ is divisible by $2$ or $3$. (b) Prove that the task is possible when $r=73$. (c) Can the task be done when $r=97$?
First we define the rectangular board in the cartesian plane with centers of the unit squares as integer coordinates and the following coordinates for the squares at the corners of $A$, $B$, $C$, $D$, as follows: $A=(1,1)$, $B=(20,1)$, $C=(20,12)$, $D=(1,12)$ Let $(x_i,y_i)$ be the coordinates of the piece after move $i$ with $(x_0,y_0)=A=(1,1)$ the initial position of the piece. $1 \le x_i \le 20$, and also $1 \le y_i \le 12$. Let $\Delta x_i = x_i-x_{i-1}$, $\Delta y_i = y_i-y_{i-1}$ Then, for any given $r$, we have $(\Delta x_i)^2+(\Delta y_i)^2=\left( \sqrt{r} \right)^2=r$ for all $i$ Part (a): In order to find out the conditions for which $r$ is divisible by 2 we are going to look at the following three cases: (1) When both $\Delta x_i$ and $\Delta y_i$ are divisible by $2$. (2) When both $\Delta x_i$ and $\Delta y_i$ are odd. (3) When one of $\Delta x_i$ and $\Delta y_i$ is even and the other one is odd. Case (1): Since $\Delta x_i \equiv 0\;(mod \; 2)$ and $\Delta y_i \equiv 0\;(mod \; 2)$, then $(\Delta x_i)^2+(\Delta y_i)^2 \equiv (0^2+0^2)\;(mod \; 2)\equiv 0\;(mod \; 2)$. Thus, for $r\equiv 0\;(mod \; 2)$, this case is a valid one. Case (2): Since $\Delta x_i \equiv 1\;(mod \; 2)$ and $\Delta y_i \equiv 1\;(mod \; 2)$, then $(\Delta x_i)^2+(\Delta y_i)^2 \equiv (1^2+1^2)\;(mod \; 2)\equiv 0\;(mod \; 2)$. Thus, for $r\equiv 0\;(mod \; 2)$, this case is a valid one. Case (3): Since $\Delta x_i \equiv 1\;(mod \; 2)$ and $\Delta y_i \equiv 0\;(mod \; 2)$, or $\Delta x_i \equiv 0\;(mod \; 2)$ and $\Delta y_i \equiv 1\;(mod \; 2)$, then $(\Delta x_i)^2+(\Delta y_i)^2 \equiv (1^2+0^2)\;(mod \; 2)\equiv 1\;(mod \; 2)\not\equiv 0\;(mod \; 2)$. Thus, for $r\equiv 0\;(mod \; 2)$, this case is NOT a valid one. Having proved that Case (1) and Case (2) are the only valid cases for $r\equiv 0\;(mod \; 2)$ we are going to see what happens for both cases when we start with a square where both coordinates are odd: if $(x_{i-1},y_{i-1}) \equiv (1,1) \;(mod \; 2)$, then for case (1): $(x_{i-1}+\Delta x_i,y_{i-1}+\Delta y_i) \equiv (1+0,1+0) \;(mod \; 2) \equiv (1,1) \;(mod \; 2)$ and for case (2): $(x_{i-1}+\Delta x_i,y_{i-1}+\Delta y_i) \equiv (1+1,1+1) \;(mod \; 2) \equiv (0,0) \;(mod \; 2)$ and $(x_{i-1}+2\Delta x_i,y_{i-1}+2\Delta y_i) \equiv (1+2,1+2) \;(mod \; 2) \equiv (1,1) \;(mod \; 2)$ This means that when $r$ is divisible by two, when starting at $(1,1)$ no matter how many moves you make all $(x_{i},y_{i})$ will either be $\equiv (0,0) \;(mod \; 2)$ or $\equiv (1,1) \;(mod \; 2)$. Since the ending coordinate is $(20,1) \equiv (0,1) \;(mod \; 2) \not\equiv (1,1) \;(mod \; 2)$ and $\not\equiv (0,0) \;(mod \; 2)$, then the task cannot be done if $r$ is divisible by $2$. Now we look at the conditions for which $r$ is divisible by 3 by looking at the following three cases: (4) When both $\Delta x_i$ and $\Delta y_i$ are divisible by $3$. (5) When one of them is not divisible by $3$ and the other one is. (6) When neither $\Delta x_i$ nor $\Delta y_i$ is divisible by $3$. Case (4): Since $\Delta x_i \equiv 0\;(mod \; 3)$ and $\Delta y_i \equiv 0\;(mod \; 3)$, then $(\Delta x_i)^2+(\Delta y_i)^2 \equiv (0^2+0^2)\;(mod \; 3)\equiv 0\;(mod \; 3)$. Thus, for $r\equiv 0\;(mod \; 3)$, this case is a valid one. Case (5): Since either $\Delta x_i$ or $\Delta y_i \equiv \pm 1\;(mod \; 3)$ and the other $\equiv 1\;(mod \; 3)$, then $(\Delta x_i)^2+(\Delta y_i)^2 \equiv ((\pm 1)^2+0^2)\;(mod \; 3)\equiv 1\;(mod \; 3)$. Thus, for $r\equiv 0\;(mod \; 3)$, this case is NOT a valid one. Case (6): Since $\Delta x_i \equiv \pm 1\;(mod \; 3)$ and $\Delta y_i \equiv \pm1 \;(mod \; 3)$, then $(\Delta x_i)^2+(\Delta y_i)^2 \equiv ((\pm 1)^2+(\pm 1)^2)\;(mod \; 3)\equiv 2\;(mod \; 3)\not\equiv 0\;(mod \; 3)$. Thus, for $r\equiv 0\;(mod \; 3)$, this case is NOT a valid one. Having proved that Case (4) is the only valid case for $r\equiv 0\;(mod \; 3)$ we are going to see what happens when we start with a square that is $\equiv (1,1)\;(mod \; 3)$ if $(x_{i-1},y_{i-1}) \equiv (1,1) \;(mod \; 3)$, then $(x_{i-1}+\Delta x_i ,y_{i-1}+\Delta y_i) \equiv (1+0,1+0) \;(mod \; 3) \equiv (1,1) \;(mod \; 3)$. Therefore starting at any $(x_{i-1},y_{i-1}) \equiv (1,1) \;(mod \; 3)$ no matter how many moves the piece will always land at another square $\equiv (1,1) \;(mod \; 3)$. Since we start at $(1,1)$, then the piece will always land at squares that are $\equiv (1,1) \;(mod \; 3)$. Since final square is $(20,1) \equiv (2,1) \;(mod \; 3) \not\equiv (1,1) \;(mod \; 3)$, then the task cannot be done if $r$ is divisible by $3$. In summary the task cannot be done if $r$ is divisible by $2$ or $r$ is divisible by $3$. Part (b): When $r=73$ we have $8^2+3^2=73$ Therefore the moves can have the following values: Since we start at $(1,1)$ and we want to end at $(20,1)$ we can write the following $11$ valid moves: Having shown that all moves comply with the possible values for $(\Delta x_i, \Delta y_i)$ and that all $(x_i, y_i)$ are inside the rectangular grid, then the task is possible when $r=73$. Part (c): When $r=97$ we have $9^2+4^2=97$ Therefore the moves can have the following values: Let $S$ be a set of the squares of the rectangle with the condition $x_i \equiv \left\lceil \frac{y_i}{4} \right\rceil \;(mod \; 2)$ for $(x_i,y_i)$. That is, the reminder of $x_i$ when divided by 2 is the same as the reminder of $\left\lceil \frac{y_i}{4} \right\rceil$ when divided by 2 where $\left\lceil k \right\rceil$ is the smallest integer that is not less than $k$. If that condition is true, then $(x_i,y_i) \in S$ Now let's look at a case (c1) where $(x_i,y_i) \in S$ and $x_i \equiv 0 \;(mod \; 2)$, thus $\left\lceil \frac{y_i}{4} \right\rceil \equiv 0 \;(mod \; 2)$: c1 subcase 1 : $(x_{i+1},y_{i+1})=(x_i \pm 9,y_i \pm 4)$: $(x_i \pm 9) \equiv (0 \pm 9)\;(mod \; 2) \equiv 1 \;(mod \; 2)$ and $\left\lceil \frac{y_i \pm 4}{4} \right\rceil \equiv \left\lceil \frac{y_i}{4} \right\rceil \pm 1 \;(mod \; 2)\equiv (0 \pm 1) \;(mod \; 2)\equiv 1 \;(mod \; 2)$ since $x_{i+1} \equiv \left\lceil \frac{y_{i+1}}{4} \right\rceil \;(mod \; 2)$, then $(x_{i+1},y_{i+1}) \in S$ for this subcase. c1 subcase 2 : $(x_{i+1},y_{i+1})=(x_i \pm 4,y_i \pm 9)$: This move is not allowed because the only values of $1 \le y_i \le 12$ such that $\left\lceil \frac{y_i}{4} \right\rceil \equiv 0 \;(mod \; 2)$ are $5,6,7,8$. Adding or subtracting 9 from any of these numbers will either make $y_{i+1}<0$ or $y_{i+1}>12$ and will put the piece outside of the board. In summary, for case (c1), $(x_{i+1},y_{i+1}) \in S$ Now let's look at a case (c2) where $(x_i,y_i) \in S$ and $x_i \equiv 1 \;(mod \; 2)$, thus $\left\lceil \frac{y_i}{4} \right\rceil \equiv 1 \;(mod \; 2)$: c2 subcase 1 : $(x_{i+1},y_{i+1})=(x_i \pm 9,y_i \pm 4)$: $(x_i \pm 9) \equiv (1 \pm 9)\;(mod \; 2) \equiv 0 \;(mod \; 2)$ and $\left\lceil \frac{y_i \pm 4}{4} \right\rceil \equiv \left\lceil \frac{y_i}{4} \right\rceil \pm 1 \;(mod \; 2)\equiv (1 \pm 1) \;(mod \; 2)\equiv 0 \;(mod \; 2)$ since $x_{i+1} \equiv \left\lceil \frac{y_{i+1}}{4} \right\rceil \;(mod \; 2)$, then $(x_{i+1},y_{i+1}) \in S$ for this subcase. c2 subcase 2 : $(x_{i+1},y_{i+1})=(x_i \pm 4,y_i \pm 9)$: $(x_i \pm 4) \equiv (1 \pm 4)\;(mod \; 2) \equiv 1 \;(mod \; 2)$ Notice that the only values of $1 \le y_i \le 12$ such that $\left\lceil \frac{y_i}{4} \right\rceil \equiv 0 \;(mod \; 2)$ are $1,2,3,4,9,10,11,12$. Adding 9 to the values of 1,2, or 3 or subtracting 9 from the values of 10, 11, or 12, will result in $\left\lceil \frac{y_{i+1}}{4} \right\rceil \equiv 1 \;(mod \; 2)$. However, subtracting 9 from the values of 1,2, or 3, or adding 9 for the values of 10,11, or 12 will put the piece outside the board and not a valid move. Furthermore, for the values of 4 and 9, adding or subtracting 9 from them will result in the piece being outside of the board and not a valid move. Therefore for all valid moves in this subcase which are 9 added to 1, 2, or 3 or 9 subtracted from 10, 11, or 12, the result will be $\left\lceil \frac{y_{i+1}}{4} \right\rceil \equiv 1 \;(mod \; 2)$ since $x_{i+1} \equiv \left\lceil \frac{y_{i+1}}{4} \right\rceil \;(mod \; 2)$, then $(x_{i+1},y_{i+1}) \in S$ for this subcase. In summary, for case (c2), $(x_{i+1},y_{i+1}) \in S$ Therefore if $(x_{i},y_{i}) \in S$ then $(x_{i+1},y_{i+1}) \in S$ Now we look at the starting point: $(1,1)$: Since $1 \equiv 1\;(mod \; 2)$ and $\left\lceil \frac{1}{4} \right\rceil \equiv 1\;(mod \; 2)$, then $(1,1) \in S$ and all subsequent squares after that $\in S$ Now let's look at the end point: $(20,1)$: Since $20 \equiv 0\;(mod \; 2)$ and $\left\lceil \frac{1}{4} \right\rceil \equiv 1\;(mod \; 2)$, then $20 \not\equiv \left\lceil \frac{1}{4} \right\rceil \;(mod \; 2)$ and $(20,1) \not\in S$ Since $(20,1) \not\in S$, then there are no valid moves starting from $(1,1)$ that will end in $(20,1)$ Therefore the task cannot be done when $r=97$. ~Tomas Diaz. orders@tomasdiaz.com Alternate solutions are always welcome. If you have a different, elegant solution to this problem, please add it to this page.
null
null
null
null
null
null
null
null
null
null
null
null
null
null
null
null
null
null
null
null
null
null
null
null
null
null
null
null
null
null
null
null
null
null
null
In the plane the points with integer coordinates are the vertices of unit squares. The squares are colored alternatively black and white (as on a chessboard). For any pair of positive integers $m$ and $n$, consider a right-angled triangle whose vertices have integer coordinates and whose legs, of lengths $m$ and $n$, lie along edges of the squares. Let $S_{1}$ be the total area of the black part of the triangle and $S_{2}$ be the total area of the white part. Let $f(m,n)=|S_{1}-S_{2}|$ (a) Calculate $f(m,n)$ for all positive integers $m$ and $n$ which are either both even or both odd. (b) Prove that $f(m,n) \le \frac{1}{2} max\left\{ m,n \right\}$ for all $m$ and $n$. (c) Show that there is no constant $C$ such that $f(m,n)<C$ for all $m$ and $n$.
For any pair of positive integers $m$ and $n$, consider a rectangle $ABCD$ whose vertices have integer coordinates and whose legs, of lengths $m$ and $n$, lie along edges of the squares. Let $A$, $B$, $C$, and $D$, be the lower left vertex, lower right vertex, upper right vertex, and upper left vertex of rectangle $ABCD$ respectively. Let $T_{1}$ be the total area of the black part of the rectangle and $T_{2}$ be the total area of the white part. Let $g(m,n)=|T_{1}-T_{2}|$ Part (a) case: $m$ and $n$ which are both even Since $m$ and $n$ are both even, the total area of the rectangle $ABCD$ is $m \times n$ which is also even Since every row has an even number of squares there are equally as many white squares than black squares for each row. Since every column has an even number of squares there are equally as many white squares than black squares for each column. This means that in the rectangle there are equal number of white squares and black squares. Therefore $T_{1}=T_{2}=\frac{mn}{2}$ and $g(m,n)=|T_{1}-T_{2}|=0$ Let $M$ be the midpoint of line $AC$. Them $M$ is at coordinate $(A_{x}+\frac{m}{2},A_{y}+\frac{n}{2})$ Since both $m$ and $n$ are even, then $M$ has integer coordinates. Starting with vertex $A$, because the length of $AB$ is even, then the color for the square inside rectangle $ABCD$ closest to $B$ is the opposite color of the square inside rectangle $ABCD$ closest to $A$, then starting with vertex $B$, because the length of $BC$ is even, then the color of the square inside rectangle $ABCD$ closest to $C$ is the opposite color of the square inside rectangle $ABCD$ closest to $B$. this means that the color of the square inside rectangle $ABCD$ closest to $A$ is the same as the color of the square inside rectangle $ABCD$ closest to $C$. Likewise, the color of the square inside rectangle $ABCD$ closest to $B$ is the same as the color of the square inside rectangle $ABCD$ closest to $D$. This color pattern and the fact that the midpoint $M$ has integer coordinates indicates that triangle $ABC$ has the same color pattern as triangle $CDA$ rotated 180 degrees. Therefore, the white area in triangle $ABC$ is the same as the white area in triangle $CDA$ and the black area in triangle $ABC$ is the same as the black area in triangle $CDA$. Thus $S_{1}=\frac{T_{1}}{2}$ and $S_{2}=\frac{T_{2}}{2}$, which gives $f(m,n)=\frac{g(m,n)}{2}=0$ Therefore $f(m,n)=0$ when both $m$ and $n$ are even. case: $m$ and $n$ which are both odd Since $m$ and $n$ are both odd, the total area of the rectangle $ABCD$ is $m \times n$ which is also odd Since the total area is odd, then $\frac{mn}{2}$ is not an integer but $\frac{mn+1}{2}$ and $\frac{mn-1}{2}$ are. This means that in the rectangle there are $\frac{mn+1}{2}$ squares of one color and $\frac{mn+1}{2}$ squares of the other color $g(m,n)=|T_{1}-T_{2}|=\left| \frac{mn+1}{2}-\frac{mn-1}{2} \right|=1$ Let $M$ be the midpoint of line $AC$. Them $M$ is at coordinate $(A_{x}+\frac{m}{2},A_{y}+\frac{n}{2})$ Since both $m$ and $n$ are odd, then $M$ has non-integer coordinates coordinates but their decimal portions are both $0.5$. This means that $M$ lies in the middle of the center square. Starting with vertex $A$, because the length of $AB$ is odd, then the color for the square inside rectangle $ABCD$ closest to $B$ is the same color of the square inside rectangle $ABCD$ closest to $A$, then starting with vertex $B$, because the length of $BC$ is odd, then the color of the square inside rectangle $ABCD$ closest to $C$ is the same color of the square inside rectangle $ABCD$ closest to $B$. this means that the color of the square inside rectangle $ABCD$ closest to $A$ is the same as the color of the square inside rectangle $ABCD$ closest to $C$. Likewise, the color of the square inside rectangle $ABCD$ closest to $B$ is the same as the color of the square inside rectangle $ABCD$ closest to $D$. This color pattern and the fact that the midpoint $M$ in in the center of the middle square that triangle $ABC$ has the same color pattern as triangle $CDA$ rotated 180 degrees. Therefore, the white area in triangle $ABC$ is the same as the white area in triangle $CDA$ and the black area in triangle $ABC$ is the same as the black area in triangle $CDA$. Thus $f(m,n)=\frac{g(m,n)}{2}=\frac{1}{2}$ Therefore $f(m,n)=\frac{1}{2}$ when both $m$ and $n$ are odd. To summarize part (a), $f(m,n)=\begin{cases} 0 & m,n\;are\;both\;even \\ \frac{1}{2} & m,n\;are\;both\;odd\\ something \; else & othwerwise\end{cases}$ Part (b) Since $f(m,n)=\begin{cases} 0 & m,n\;are\;both\;even \\ \frac{1}{2} & m,n\;are\;both\;odd\\ something \; else & othwerwise\end{cases}$ then for these cases the minimum values that $m$ and $n$ can have are 1. So for the cases where both are odd or both are even $f(m,n) \le \frac{1}{2} max\left\{ m,n \right\}$ with equality at $m=n=1$ Now we need to find the case were one of them is odd and the other one is even. case $max\left\{ m,n \right\}$ is odd and $min\left\{ m,n \right\}$ is even, means that $max\left\{ m,n \right\}-1$ is even Therefore $f\left( max\left\{ m,n \right\}-1,min\left\{ m,n \right\} \right)=0$ $f(m,n)=f\left( max\left\{ m,n \right\}-1,min\left\{ m,n \right\} \right)+h(1,min\left\{ m,n \right\})$, where $h(1,n)$ is the absolute value of the difference between the white area and black area of a triangle with base or size 1 and height n where this triangle is not necessarily a rectangular triangle. Therefore $h(1,n) \le$ area of such triangle. Thus, $h(1,min\left\{ m,n \right\}) \le \frac{1}{2}min\left\{ m,n \right\}$ $f\left( max\left\{ m,n \right\}-1,min\left\{ m,n \right\} \right)+h(1,min\left\{ m,n \right\}) \le 0 +\frac{1}{2}min\left\{ m,n \right\}$ $f(m,n) \le \frac{1}{2}min\left\{ m,n \right\}$ when $max\left\{ m,n \right\}$ is odd and $min\left\{ m,n \right\}$ is even Which also means that $f(m,n) \le \frac{1}{2}max\left\{ m,n \right\}$ when $max\left\{ m,n \right\}$ is odd and $max\left\{ m,n \right\}$ is even Now we look at the case where $max\left\{ m,n \right\}$ is even and $min\left\{ m,n \right\}$ is odd, means that $min\left\{ m,n \right\}-1$ is even Therefore $f\left( min\left\{ m,n \right\}-1,max\left\{ m,n \right\} \right)=0$ $f(m,n)=f\left( min\left\{ m,n \right\}-1,max\left\{ m,n \right\} \right)+h(1,max\left\{ m,n \right\})$, Thus, $h(1,max\left\{ m,n \right\}) \le \frac{1}{2}max\left\{ m,n \right\}$ $f\left( min\left\{ m,n \right\}-1,max\left\{ m,n \right\} \right)+h(1,max\left\{ m,n \right\}) \le 0 +\frac{1}{2}max\left\{ m,n \right\}$ $f(m,n) \le \frac{1}{2}max\left\{ m,n \right\}$ when $max\left\{ m,n \right\}$ is even and $min\left\{ m,n \right\}$ is odd Therefore, for all four cases: $m$ and $n$ are both odd; $m$ and $n$ are both even; $max\left\{ m,n \right\}$ is odd with $min\left\{ m,n \right\}$ is even; and $max\left\{ m,n \right\}$ is even with $min\left\{ m,n \right\}$ is odd we have: $f(m,n) \le \frac{1}{2}max\left\{ m,n \right\}$ with equality at $m=n=1$ Part (c) Consider the case where $m=k$ and $n=k+1$ and k is even. $f(m,n)=f(k,k+1)=f(k,k)+h(k,1)=h(k,1)$ We're going to calculate $h(k,1)$ noticing that the region for $h(k,1)$ in $f(m,n)$ will be the triangle formed by lines $y=kx$, $y=\frac{k+1}{k}x$ and $x=k$. So, to calculate $h(k,1)$ we first calculate the total area of the black triangular regions within the triangle (assuming the first corner is white) with the following series formula: $S_{1}=\sum_{j=1}^{k}\frac{1}{2}\left( \frac{k+1}{k}j-j \right)\left(j- \frac{k}{k+1}j \right)=\sum_{j=1}^{k}\frac{1}{2}\left( \frac{k+1-k}{k} \right)\left( \frac{k+1-k}{k+1} \right)j^{2}$ $S_{1}=\frac{1}{2k(k+1)}\sum_{j=1}^{k}j^{2}=\frac{1}{2k(k+1)}\frac{k(k+1)(2k+1)}{6}=\frac{2k+1}{12}$ Then, $S_{2}=\frac{k}{2}-S_{1}=\frac{k}{2}-\frac{2k+1}{12}=\frac{4k-1}{12}$ $h(k,1)=|S_{2}-S_{1}|=\left| \frac{4k-1}{12}-\frac{2k+1}{12} \right|=\frac{k-1}{6}$ Thus for even $k$ we have $f(k,k+1)=h(k,1)=\frac{k-1}{6}$ Since for this case, $C=\lim_{k \to \infty} f(k,k+1)=\lim_{k \to \infty} \frac{k-1}{6}=\infty$, then there is no upper bound for this case. Since part (c) describes an upper bound for all $m$ and $n$, then having found one case where there is no upper bound means that there is no upper bound when considering all $m$ and $n$ Therefore, there is no constant $C$ such that $f(m,n)<C$ for all $m$ and $n$ because $C \to \infty\;$ on some cases. ~ Tomas Diaz. orders@tomasdiaz.com Alternate solutions are always welcome. If you have a different, elegant solution to this problem, please add it to this page.
null
null
null
null
null
null
null
null
null
null
null
null
null
null
null
null
null
null
null
null
null
null
null
null
null
null
null
null
null
null
null
null
null
null
null
In the convex quadrilateral $ABCD$, the diagonals $AC$ and $BD$ are perpendicular and the opposite sides $AB$ and $DC$ are not parallel. Suppose that the point $P$, where the perpendicular bisectors of $AB$ and $DC$ meet, is inside $ABCD$. Prove that $ABCD$ is a cyclic quadrilateral if and only if the triangles $ABP$ and $CDP$ have equal areas.
This problem needs a solution. If you have a solution for it, please help us out by adding it.
null
null
null
null
null
null
null
null
null
null
null
null
null
null
null
null
null
null
null
null
null
null
null
null
null
null
null
null
null
null
null
null
null
null
null
Determine all finite sets $S$ of at least three points in the plane which satisfy the following condition: For any two distinct points $A$ and $B$ in $S$, the perpendicular bisector of the line segment $AB$ is an axis of symmetry of $S$.
Upon reading this problem and drawing some points, one quickly realizes that the set $S$ consists of all the vertices of any regular polygon. Now to prove it with some numbers: Let $S=\left\{ P_{0},P_{1},P_{2},...,P_{n-1} \right\}$, with $n\ge 3$, where $P_{i}$ is a vertex of a polygon which we can define their $xy$ coordinates as: $P_{i}=\left\langle Rcos\left( \frac{2\pi}{n}i \right),Rsin\left( \frac{2\pi}{n}i \right) \right\rangle$ for $i=0,1,2,...,(n-1)$. That defines the vertices of any regular polygon with $R$ being the radius of the circumcircle of the regular $n$-sided polygon. Now we can pick any points $A$ and $B$ of the set as: $A=P_{a}$ and $B=P_{b}$, where $a=0,1,2,...,(n-1)$; $b=0,1,2,...,(n-1)$; and $a\ne b$ Then, $A=\left\langle Rcos\left( \frac{2\pi}{n}a \right),Rsin\left( \frac{2\pi}{n}a \right) \right\rangle$ and $B=\left\langle Rcos\left( \frac{2\pi}{n}b \right),Rsin\left( \frac{2\pi}{n}b \right) \right\rangle$ Let $O$ be point $(0,0)$ which is not part of $S$ Then, $\angle P_{0}OA = \frac{2\pi}{n}a$, and $\angle P_{0}OB = \frac{2\pi}{n}b$ The perpendicular bisector of $AB$ passes through $O$. Let point $M_{AB}$, not in $S$ be a point that passes through the perpendicular bisector of $AB$ at a distance $R$ from $O$ Then, $\angle P_{0}OM_{AB} =\frac{2\pi}{n}\frac{a+b}{2}$ and $M_{AB}=\left\langle Rcos\left( \frac{2\pi}{n}\frac{a+b}{2} \right),Rsin\left( \frac{2\pi}{n}\frac{a+b}{2} \right) \right\rangle$ CASE I: $a+b$ is even $k=\frac{a+b}{2}$ and $k$ is integer Then $M_{AB}=\left\langle Rcos\left( \frac{2\pi}{n}k \right),Rsin\left( \frac{2\pi}{n}k \right) \right\rangle=P_{k}$ This means that the perpendicular bisector also passes through a point $P_{k}$ of $S$ Let $c$ be any positive integer $\angle P_{k}OP_{(k+c)\; mod\; n}=\frac{2\pi}{n}\left( (k+c-k)\; mod\; n \right)=\frac{2\pi}{n}\left( c\; mod\; n \right)$ and $\angle P_{k}OP_{(k-c)\; mod\; n}=\frac{2\pi}{n}\left( (k-(k-c))\; mod\; n \right)=\frac{2\pi}{n}\left( c\; mod\; n \right)$ Therefore, $\angle P_{k}OP_{(k+c)\; mod\; n}=\angle P_{k}OP_{(k-c)\; mod\; n}$ for any integer $c$. Also, since $\left| OP_{(k+c)\; mod\; n} \right|=\left| OP_{(k-c)\; mod\; n} \right|=R$ for any integer $c$ then this proves that the bisector of any points $A$ and $B$ is an axis of symmetry for this case. CASE II: $a+b$ is odd $k=\frac{a+b+1}{2}$ and $k$ is integer $m=\frac{a+b-1}{2}$ and $m$ is integer Then $M_{AB}=\left\langle Rcos\left( \frac{2\pi}{n}\frac{a+b}{2} \right),Rsin\left( \frac{2\pi}{n}\frac{a+b}{2} \right) \right\rangle$ This means that the perpendicular bisector does not pass through any point of $S$, but their closest points are $P_{k}$ and $P_{m}$ and that $\angle MOP_{k}=\angle MOP_{m}=\frac{\pi}{n}$ Let $c$ be any positive integer $\angle P_{k}OP_{(k+c)\; mod\; n}=\frac{2\pi}{n}\left( (k+c-k)\; mod\; n \right)=\frac{2\pi}{n}\left( c\; mod\; n \right)$ $\angle MOP_{(k+c)\; mod\; n}=\angle MOP_{k}+\angle P_{k}OP_{(k+c)\; mod\; n}=\frac{\pi}{n}+\frac{2\pi}{n}\left( c\; mod\; n \right)$ and $\angle P_{m}OP_{(m-c)\; mod\; n}=\frac{2\pi}{n}\left( (m-(m-c))\; mod\; n \right)=\frac{2\pi}{n}\left( c\; mod\; n \right)$ $\angle MOP_{(m-c)\; mod\; n}=\angle MOP_{m}+\angle P_{m}OP_{(m-c)\; mod\; n}=\frac{\pi}{n}+\frac{2\pi}{n}\left( c\; mod\; n \right)$ Therefore, $\angle MOP_{(k+c)\; mod\; n}=\angle MOP_{(m-c)\; mod\; n}$ for any integer $c$. Since $m=k-1$, $\angle MOP_{(k+c)\; mod\; n}=\angle MOP_{(k-1-c)\; mod\; n}$ Also, since $\left| OP_{(k+c)\; mod\; n} \right|=\left| OP_{(m-c)\; mod\; n} \right|=R$ for any integer $c$ then this proves that the bisector of any points $A$ and $B$ is an axis of symmetry for this case. Having proven both cases, then the set $S$ of points that comply with the given condition is the set of the vertices of any regular polygon of any number of sides. ~Tomas Diaz. orders@tomasdiaz.com Alternate solutions are always welcome. If you have a different, elegant solution to this problem, please add it to this page.
null
null
null
null
null
null
null
null
null
null
null
null
null
null
null
null
null
null
null
null
null
null
null
null
null
null
null
null
null
null
null
null
null
null
null
Two circles $G_1$ and $G_2$ intersect at two points $M$ and $N$. Let $AB$ be the line tangent to these circles at $A$ and $B$, respectively, so that $M$ lies closer to $AB$ than $N$. Let $CD$ be the line parallel to $AB$ and passing through the point $M$, with $C$ on $G_1$ and $D$ on $G_2$. Lines $AC$ and $BD$ meet at $E$; lines $AN$ and $CD$ meet at $P$; lines $BN$ and $CD$ meet at $Q$. Show that $EP=EQ$.
$\textbf{Proof of problem:}$ Let ray $NM$ intersect $AB$ at $X$. By our lemma, $\textit{(the two circles are tangent to AB)}$, $X$ bisects $AB$. Since $\triangle{NAX}$ and $\triangle{NPM}$ are similar, and $\triangle{NBX}$ and $\triangle{NQM}$ are similar implies $M$ bisects $PQ$. By simple parallel line rules, $\angle{ABM}=\angle{EBA}$. Similarly, $\angle{BAM}=\angle{EAB}$, so by $\textit{ASA}$ criterion, $\triangle{ABM}$ and $\triangle{EAB}$ are congruent. Now, $\angle{EBA} = \angle{ABM} = \angle{BDM}$ and $\angle{ABM} = \angle{BMD}$ since $CD$ is parallel to $AB$. But $AB$ is tangent to the circumcircle of $\triangle{BMD}$ hence $\angle{ABM} = \angle{BDM}$ and that implies $\angle{BMD} = \angle{BDM} .$So$\triangle{BMD}$ is isosceles and $\angle{BMD}=\angle{BDM}$ . Join points $E$ and $M$, $EM$ is perpendicular on $PQ$ (why?), previously we proved $MP = MQ$, hence $\triangle{EPQ}$ is isoscles and $EP = EQ$ .
null
null
null
null
null
null
null
null
null
null
null
null
null
null
null
null
null
null
null
null
null
null
null
null
null
null
null
null
null
null
null
null
null
null
null
Consider an acute triangle $\triangle ABC$. Let $P$ be the foot of the altitude of triangle $\triangle ABC$ issuing from the vertex $A$, and let $O$ be the circumcenter of triangle $\triangle ABC$. Assume that $\angle C \geq \angle B+30^{\circ}$. Prove that $\angle A+\angle COP < 90^{\circ}$.
Take $D$ on the circumcircle with $AD \parallel BC$. Notice that $\angle CBD = \angle BCA$, so $\angle ABD \ge 30^\circ$. Hence $\angle AOD \ge 60^\circ$. Let $Z$ be the midpoint of $AD$ and $Y$ the midpoint of $BC$. Then $AZ \ge R/2$, where $R$ is the radius of the circumcircle. But $AZ = YP$ (since $AZYP$ is a rectangle). Now $O$ cannot coincide with $Y$ (otherwise $\angle A$ would be $90^\circ$ and the triangle would not be acute-angled). So $OP > YP \ge R/2$. But $PC = YC - YP < R - YP \le R/2$. So $OP > PC$. Hence $\angle COP < \angle OCP$. Let $CE$ be a diameter of the circle, so that $\angle OCP = \angle ECB$. But $\angle ECB = \angle EAB$ and $\angle EAB + \angle BAC = \angle EAC = 90^\circ$, since $EC$ is a diameter. Hence $\angle COP + \angle BAC < 90^\circ$.
Notice that because $\angle{PCO} = 90^\circ - \angle{A}$, it suffices to prove that $\angle{POC} < \angle{PCO}$, or equivalently $PC < PO.$ Suppose on the contrary that $PC > PO$. By the triangle inequality, $2 PC = PC + PC > PC + PO > CO = R$, where $R$ is the circumradius of $ABC$. But the Law of Sines and basic trigonometry gives us that $PC = 2R \sin B \cos C$, so we have $4 \sin B \cos C > 1$. But we also have $4 \sin B \cos C \le 4 \sin B \cos (B + 30^\circ) = 2 (\sin (2B + 30^\circ) - \sin 30^\circ) \le 2 (1 - \frac{1}{2}) = 1$ because $\angle{C} \ge \angle{B} + 30^\circ$, and so we have a contradiction. Hence $PC < PO$ and so $\angle{PCO} + \angle{A} < 90^\circ$, as desired.
null
null
null
null
null
null
null
null
null
null
null
null
null
null
null
null
null
null
null
null
null
null
null
null
null
null
null
null
null
null
null
null
null
null
$S$ is the set of all $(h,k)$ with $h,k$ non-negative integers such that $h + k < n$. Each element of $S$ is colored red or blue, so that if $(h,k)$ is red and $h' \le h,k' \le k$, then $(h',k')$ is also red. A type $1$ subset of $S$ has $n$ blue elements with different first member and a type $2$ subset of $S$ has $n$ blue elements with different second member. Show that there are the same number of type $1$ and type $2$ subsets.
This problem needs a solution. If you have a solution for it, please help us out by adding it.
null
null
null
null
null
null
null
null
null
null
null
null
null
null
null
null
null
null
null
null
null
null
null
null
null
null
null
null
null
null
null
null
null
null
null
Let $ABC$ be an acute-angled triangle with $AB\neq AC$. The circle with diameter $BC$ intersects the sides $AB$ and $AC$ at $M$ and $N$ respectively. Denote by $O$ the midpoint of the side $BC$. The bisectors of the angles $\angle BAC$ and $\angle MON$ intersect at $R$. Prove that the circumcircles of the triangles $BMR$ and $CNR$ have a common point lying on the side $BC$.
Let $\angle{ACB} = a$, $\angle{CBA} = b$, and $\angle{ONR} = k$. Call $\omega$ the circle with diameter $BC$ and $\odot{AMN}$ the circumcircle of $\triangle{AMN}$. Our ultimate goal is to show that $\angle{CNR} + \angle{RMB} = 180^\circ$. To show why this solves the problem, assume this statement holds true. Call $K$ the intersection point of the circumcircle of $\triangle{CNR}$ with side $BC$. Then, $\angle{RKC} = 180^\circ - \angle{CNR}$, and $\angle{RKB} = \angle{CNR}$. Since $\angle{RMB} = 180^\circ - \angle{CNR}$, $\angle{RKB} + \angle{RMB} = 180^\circ$, implying $K$ also lies on the circumcircle of $\triangle{BMR}$, thereby solving the problem. We now prove that $\angle{CNR} + \angle{RMB} = 180^\circ$. Note that $ON$ and $OM$ are radii of $\omega$, so $\triangle{MON}$ is isosceles. The bisector of $\angle{MON}$ is thus the perpendicular bisector of $MN$. Since $R$ lies on the bisector of $\angle{MON}$, $\angle{ONR} = \angle{RMO} = k$. Angle computations yield that from $\angle{AMN} = 180^\circ - \angle{BMN} = 180^\circ - (180^\circ - \angle{ACB}) = \angle{ACB}$ and from $\angle{BMO} = \angle{MBO} =$ $\angle{ABC}$. The bisector of $\angle{BAC}$ hits $\odot{AMN}$ at the midpoint of the arc $MN$ not containing $A$. This point must lie on the perpendicular bisector of segment $MN$, which is the bisector of $\angle{MON}$. It follows that $R$ is indeed the midpoint of arc $MN$, so $A$, $M$, $R$, $N$, are concyclic. Since $\angle{RAN}$ and $\angle{RMN}$ subtend the same arc $NR$, $\angle{RAN}$ = $\angle{RMN}$. With $AR$ being the bisector of $\angle{BAC}$, we have We know that $\angle{RMN} = 180^\circ - b - k - a$. so we have $180^\circ - b - k - a = 90^\circ - \tfrac{a}{2} - \tfrac{b}{2}$. Since $\angle{CNR} = \angle{CNO} + \angle{ONR} = a + k$, and $\angle{RMB} = \angle{OMB} + \angle{RMO} = b + k$, we have The problem is solved. $\textbf{NOTE:}$ We have $\angle{RKB} + \angle{KBA} + \angle{BAK} = 180^\circ \implies \angle{BAK} = 180^\circ - (180^\circ - \angle{RMB}) - \angle{KBA}$. Noting that $180^\circ - \angle{RMB} = 180^\circ - b - k = 180^\circ - (90^\circ - \tfrac{a}{2} + \tfrac{b}{2}) = 90^\circ + \tfrac{a}{2} - \tfrac{b}{2}$, we then have which is indeed the measure of $\angle{BAR}$. This implies that $K$ lies on the bisector of $\angle{BAC}$, and from this, it is clear that $K$ must lie on the interior of segment $BC$. Not proving that $K$ had to lie in the interior of $BC$ was a reason that a large portion of students who submitted a solution received a 1-point deduction.
null
null
null
null
null
null
null
null
null
null
null
null
null
null
null
null
null
null
null
null
null
null
null
null
null
null
null
null
null
null
null
null
null
null
null
Six points are chosen on the sides of an equilateral triangle $ABC$: $A_1, A_2$ on $BC$, $B_1$, $B_2$ on $CA$ and $C_1$, $C_2$ on $AB$, such that they are the vertices of a convex hexagon $A_1A_2B_1B_2C_1C_2$ with equal side lengths. Prove that the lines $A_1B_2, B_1C_2$ and $C_1A_2$ are concurrent.
This problem needs a solution. If you have a solution for it, please help us out by adding it.
null
null
null
null
null
null
null
null
null
null
null
null
null
null
null
null
null
null
null
null
null
null
null
null
null
null
null
null
null
null
null
null
null
null
null
Let $ABC$ be triangle with incenter $I$. A point $P$ in the interior of the triangle satisfies $\angle PBA+\angle PCA = \angle PBC+\angle PCB$. Show that $AP \geq AI$, and that equality holds if and only if $P=I.$
We have and similarly Since $\angle PBA + \angle PCA = \angle PBC + \angle PCB$, we have $\angle PCB - \angle PCA = \angle PBA - \angle PBC.$ It follows that Hence, $B,P,I,$ and $C$ are concyclic. Let ray $AI$ meet the circumcircle of $\triangle ABC\,$ at point $J$. Then, by the Incenter-Excenter Lemma, $JB=JC=JI=JP$. Finally, $AP+JP \geq AJ = AI+IJ$ (since triangle APJ can be degenerate, which happens only when $P=I$), but $JI=JP$; hence $AP \geq AI$ and we are done. By Mengsay LOEM , Cambodia IMO Team 2015 latexed by tluo5458 :) minor edits by lpieleanu
null
null
null
null
null
null
null
null
null
null
null
null
null
null
null
null
null
null
null
null
null
null
null
null
null
null
null
null
null
null
null
null
null
null
null
Real numbers $a_1, a_2, \dots , a_n$ are given. For each $i$ ($1\le i\le n$) define and let . (a) Prove that, for any real numbers $x_1\le x_2\le \cdots\le x_n$, (b) Show that there are real numbers $x_1\le x_2\le x_n$ such that equality holds in (*)
Since $d_i=\max\{a_j:1\le j\le i\}-\min\{a_j:i\le j\le n\}$, all $d_i$ can be expressed as $a_p-a_q$, where $1\le p\le i\le q \le n$. Thus, $d$ can be expressed as $a_p-a_q$ for some $p$ and $q$, $1\le p\le q\le n$ Lemma) $d\ge 0$ Assume for contradiction that $d<0$, then for all $i$, $a_i \le \max\{a_j:1\le j\le i\}\le \min\{a_j:i\le j\le n\}\le a_{i+1}$ $a_i\le a_{i+1}$ Then, ${a_i}$ is a non-decreasing function, which means, $\max\{a_j:1\le j\le i\}=a_i$, and $\min\{a_j:i\le j\le n\}\le a_{i+1}=a_i$, which means, ${d_i}={0}$. Then, $d=0$ and contradiction. a) Case 1) $d=0$ If $d=0$, $\max\{|x_i-a_i|:1\le i\le n\}$ is the maximum of a set of non-negative number, which must be at least $0$. Case 2) $d>0$ (We can ignore $d<0$ because of lemma) Using the fact that $d$ can be expressed as $a_p-a_q$ for some $p$ and $q$, $1\le p\le q\le n$. $x_p\le x_q$ Assume for contradiction that $\max\{|x_i-a_i|:1\le i\le n\}<\dfrac{d}{2}$. Then, $\forall x_i$, $|x_i-a_i|<\dfrac{d}{2}$. $|x_p-a_p|<\dfrac{d}{2}$, and $|x_q-a_q|<\dfrac{d}{2}$ Thus, $x_p>a_p-\dfrac{d}{2}$ and $x_q<a_q+\dfrac{d}{2}$. Subtracting the two inequality, we will obtain: $x_p>x_q$ --- contradiction ($p\le q \rightarrow x_p\le x_q$). Thus, $\max\{|x_i-a_i|:1\le i\le n\}\ge\dfrac{d}{2}$ (b) A set of ${x_i}$ where the equality in (*) holds is: Since $\max\{a_j:1\le j\le i\}$ is a non-decreasing function, $x_i$ is non-decreasing. $\forall x_i$ : Let $a_m=\max\{a_j:1\le j\le i\}$, $a_m-a_i<a_m-\min\{a_j:i\le j\le n\}=d_i$. Thus, $0\le a_m-a_i \le d$ ($0\le a_m-a_i$ because $a_m$ is the max of a set including $a_i$) $|x_i-a_i|=\left|a_m-\dfrac{d}{2}-a_i\right|=\left|(a_m-a_i)\dfrac{d}{2}\right|$ $0\le a_m-a_i\le d$ $-\dfrac{d}{2} \le (a_m-a_i)\dfrac{d}{2} \le \dfrac{d}{2}$ $\left|(a_m-a_i)-\dfrac{d}{2}\right|=|x_i-a_i|\le \frac{d}{2}$ Since $\max\{|x_i-a_i|:1\le i\le n\}\ge\dfrac{d}{2}$ and $|x_i-a_i|\le \frac{d}{2}$ $\forall x_i$, $\max\{|x_i-a_i|:1\le i\le n\}=\dfrac{d}{2}$ This is written by Mo Lam--- who is a horrible proof writer, so please fix the proof for me. Thank you. O, also the formatting. --> Alternate solutions are always welcome. If you have a different, elegant solution to this problem, please add it to this page.
null
null
null
null
null
null
null
null
null
null
null
null
null
null
null
null
null
null
null
null
null
null
null
null
null
null
null
null
null
null
null
null
null
null
null
An acute-angled triangle $ABC$ has orthocentre $H$. The circle passing through $H$ with centre the midpoint of $BC$ intersects the line $BC$ at $A_1$ and $A_2$. Similarly, the circle passing through $H$ with centre the midpoint of $CA$ intersects the line $CA$ at $B_1$ and $B_2$, and the circle passing through $H$ with centre the midpoint of $AB$ intersects the line $AB$ at $C_1$ and $C_2$. Show that $A_1$, $A_2$, $B_1$, $B_2$, $C_1$, $C_2$ lie on a circle.
Let $M_A$, $M_B$, and $M_C$ be the midpoints of sides $BC$, $CA$, and $AB$, respectively. It's not hard to see that $M_BM_C\parallel BC$. We also have that $AH\perp BC$, so $AH \perp M_BM_C$. Now note that the radical axis of two circles is perpendicular to the line connecting their centers. We know that $H$ is on the radical axis of the circles centered at $M_B$ and $M_C$, so $A$ is too. We then have $AC_1\cdot AC_2=AB_2\cdot AB_1\Rightarrow \frac{AB_2}{AC_1}=\frac{AC_2}{AB_1}$. This implies that $\triangle AB_2C_1\sim \triangle AC_2B_1$, so $\angle AB_2C_1=\angle AC_2B_1$. Therefore $\angle C_1B_2B_1=180^{\circ}-\angle AB_2C_1=180^{\circ}-\angle AC_2B_1$. This shows that quadrilateral $C_1C_2B_1B_2$ is cyclic. Note that the center of its circumcircle is at the intersection of the perpendicular bisectors of the segments $C_1C_2$ and $B_1B_2$. However, these are just the perpendicular bisectors of $AB$ and $CA$, which meet at the circumcenter of $ABC$, so the circumcenter of $C_1C_2B_1B_2$ is the circumcenter of triangle $ABC$. Similarly, the circumcenters of $A_1A_2B_1B_2$ and $C_1C_2A_1A_2$ are coincident with the circumcenter of $ABC$. The desired result follows.
null
null
null
null
null
null
null
null
null
null
null
null
null
null
null
null
null
null
null
null
null
null
null
null
null
null
null
null
null
null
null
null
null
null
null
Let $n$ be a positive integer and let $a_1,\ldots,a_k (k\ge2)$ be distinct integers in the set $\{1,\ldots,n\}$ such that $n$ divides $a_i(a_{i+1}-1)$ for $i=1,\ldots,k-1$. Prove that $n$ doesn't divide $a_k(a_1-1)$. Author: Ross Atkins, Australia
Let $n=pq$ such that $p\mid a_1$ and $q\mid a_2-1$. Suppose $n$ divides $a_k(a_1-1)$. Note $q\mid a_2-1$ implies $(q,a_2)=1$ and hence $q\mid a_3-1$. Similarly one has $q\mid a_i-1$ for all $i$'s, in particular, $p\mid a_1$ and $q\mid a_1-1$ force $(p,q)=1$. Now $(p,a_1-1)=1$ gives $p\mid a_k$, similarly one has $p\mid a_i$ for all $i$'s, that is $a_i$'s satisfy $p\mid a_i$ and $q\mid a_i-1$, but there should be at most one such integer satisfies them within the range of $1,2,\ldots,n$ for $n=pq$ and $(p,q)=1$. A contradiction!!!
Let $n = p_1^{b_1}p_2^{b_2} \cdots p_s^{b_s}$. Then after toying around with the $p_i^{b_i}$ and what they divide, we have that $p_i^{b_i} \nmid a_k$, and so in particular, $n \nmid a_k$. Assume by way of contradiction that $n \mid a_k(a_1 - 1)$. Then $n \mid a_1 - 1$. Now we shift our view towards the $a_i(a_{i + 1} - 1)$. Here each $p_i^{b_i}$ divides $a_i(a_{i + 1} - 1) \implies a_ia_{i + 1} \equiv a_i \pmod{p_i^{b_i}}$. Hence we have the chain of equivalences $a_1a_2 \equiv a_1 \pmod{p_i^{b_i}}, a_2a_3 \equiv a_2 \pmod{p_i^{b_i}}, \dots, a_{k - 1}a_k \equiv a_{k - 1} \pmod {p_i^{b_i}}$. Now we also have that $p_i^{b_i} \mid n \mid a_1 - 1$. Thus $a_1 \equiv 1 \pmod{p_i^{b_i}}$. Now plugging this value of $a_1$ modulo $p_i^{b_i}$, we obtain that $a_1 \equiv a_2 \equiv a_3 \equiv \cdots a_k \equiv 1 \pmod{p_i^{b_i}}$. Hence this chain of congruences is also true for $n$ as $p_i$ was arbitrary. However as all the $a_i \in \{1, 2, \dots, n\}$ we have that not all the $a_i$ are distinct, and so this is a contradiction.
null
null
null
null
null
null
Put $x=y=0$. Then $f(0)=0$ or $\lfloor f(0) \rfloor=1$. $\bullet$ If $\lfloor f(0) \rfloor=1$, putting $y=0$ we get $f(x)=f(0)$, that is f is constant. Substituing in the original equation we find $f(x)=0, \ \forall x \in \mathbb{R}$ or $f(x)=a, \ \forall x \in \mathbb{R}$, where $a \in [1,2)$. $\bullet$ If $f(0)=0$, putting $x=y=1$ we get $f(1)=0$ or $\lfloor f(1) \rfloor=1$. For $f(1)=0$, we set $x=1$ to find $f(y)=0 \ \forall y$, which is a solution. For $\lfloor f(1) \rfloor=1$, setting $y=1$ yields $f(\lfloor x \rfloor)=f(x), \ (*)$. Putting $x=2, y=\frac{1}{2}$ to the original we get $f(1)=f(2)\lfloor f(\frac{1}{2}) \rfloor$. However, from $(*)$ we have $f(\frac{1}{2})=f(0)=0$, so $f(1)=0$ which contradicts the fact $\lfloor f(1) \rfloor=1$. So, $f(x)=0, \ \forall x$ or $f(x)=a, \ \forall x, \ a \in [1,2)$. ( By socrates[1])
null
null
null
null
null
null
null
null
null
null
null
null
null
null
null
null
null
null
Let $y=0$, then $f(0)=f(x)\left\lfloor f(0)\right\rfloor$. Case 1: $\left\lfloor f(0)\right\rfloor\neq 0$ Then $f(x)=\frac{f(0)}{\left\lfloor f(0)\right\rfloor}$ is a constant. Let $f(x)=k$, then $k=k\left\lfloor k \right\rfloor \Leftrightarrow k=0 \vee 1\leq k<2$. It is easy to check that this are solutions. Case 2: $\left\lfloor f(0)\right\rfloor= 0$ In this case we conclude that $\left\lfloor f(0)\right\rfloor= 0\Rightarrow f(0)=0$ Lemma:If $y$ is such that $0\leq f(y)<1$, $f(y)=0$ Proof of the Lemma: If $x=1$ we have that $f(\left\lfloor x\right\rfloor y)=f(y)=f(x)\left\lfloor f(y)\right\rfloor =0$, as desired. Let $0\leq x<1$, so that we have: $0=f(0)=f(\left\lfloor x\right\rfloor y)=f(x)\left\lfloor f(y)\right\rfloor\Rightarrow$ $\Rightarrow f(x)=0 \vee 0\leq f(y)<1 \Rightarrow f(x)=0 \vee f(y)=0$, using the lemma. If $f$ is not constant and equal to $0$, letting $y$ be such that $f(y)\neq 0$ implies that $f(x)=0, \forall 0\leq x<1$. Now it's enough to notice that any real number $x$ is equal to $ky$, where $k\in \mathbb{Z}$ and $0\leq y< 1$, so that $f(x)=f(ky)=f(\left\lfloor k\right\rfloor y)=f(k)\left\lfloor f(y)\right\rfloor=0$. Since $x$ was arbitrary, we have that $f$ is constant and equal to $0$. We conclude that the solutions are $f(x)=k$, where $k=0 \vee 1\leq k<2$.( By Jorge Miranda [3] )
null
null
null
null
null
null
Clearly $f(\left\lfloor x\right\rfloor y) = f(\left\lfloor \lfloor x \rfloor \right\rfloor y) = f(\lfloor x \rfloor)\left\lfloor f(y)\right\rfloor$, so $(f(x) - f(\lfloor x \rfloor))\left\lfloor f(y)\right\rfloor = 0$ for all $x,y\in\mathbb{R}$. If $\left\lfloor f(y)\right\rfloor = 0$ for all $y \in \mathbb{R}$, then by taking $x=1$ we get $f(y)=f(1)\left\lfloor f(y)\right\rfloor = 0$, so $f$ is identically null (which checks). If, contrariwise, $\left\lfloor f(y_0)\right\rfloor \neq 0$ for some $y_0 \in \mathbb{R}$, it follows $f(x) = f(\lfloor x \rfloor)$ for all $x \in \mathbb{R}$. Now it immediately follows $f(x) = f(\lfloor x \rfloor \cdot 1) = f(x)\lfloor f(1) \rfloor$, hence $f(x)(1 - \lfloor f(1) \rfloor) = 0$. For $x=y_0$ this implies $\lfloor f(1) \rfloor = 1$. Assume $\lfloor f(0) \rfloor=0$; then $1 \leq f(1) = f\left ( 2\cdot \dfrac {1} {2} \right ) = f(2)\left \lfloor f \left ( \dfrac {1} {2} \right ) \right \rfloor = f(2)\left \lfloor f \left ( \left \lfloor \dfrac {1} {2} \right \rfloor \right ) \right \rfloor = f(2)\left \lfloor f(0) \right \rfloor = 0$, absurd. Therefore $\lfloor f(0) \rfloor \neq 0$, and now $y=0$ in the given functional equation yields $f(0) = f(x)\lfloor f(0) \rfloor$ for all $x \in \mathbb{R}$, therefore $f(x) = c \neq 0$ constant, with $\lfloor c \rfloor = \lfloor f(1) \rfloor = 1$, i.e. $c \in [1,2)$ (which obviously checks).( By mavropnevma [4])
null
Find all function $f:\mathbb{R}\rightarrow\mathbb{R}$ such that for all $x,y\in\mathbb{R}$ the following equality holds $f(\left\lfloor x\right\rfloor y)=f(x)\left\lfloor f(y)\right\rfloor$ where $\left\lfloor a\right\rfloor$ is greatest integer not greater than $a.$ Author: Pierre Bornsztein, France
null
Substituting $y=0$ we have $f(0) = f(x) [f(0)]$. If $[f0)] \ne 0$ then $f(x) = \frac{f(0)}{[f(0)]}$. Then $f(x)$ is constant. Let $f(x)=c$. Then substituting that in (1) we have $c=c[c] \Rightarrow c(1-[c])=0 \Rightarrow c=0$, or $[c]=1$. Therefore $f(x)=c$ where $c=0$ or $c \in [1,2)$ If $[f(0)] = 0$ then $f(0)=0$. Now substituting $x=1$ we have $f(y)=f(1)[f(y)]$. If $f(1) \ne 0$ then $[f(y)] = \frac{f(y)}{f(1)}$ and substituting this in (1) we have $f([x]y)=\frac{f(x)f(y)}{f(1)}$. Then $f([x]y)=f(x[y])$. Substituting $x=1/2, y=2$ we get $f(0)=f(1)$. Then $f(1)=0$, which is a contradiction Therefore $f(1)=0$. and then $f(y)=0$ for all $R$ Then the only solutions are $f(x)=0$ or $f(x)=c$ where $c \in [1,2)$.( By m.candales [2])
null
null
null
null
null
null
Firstly, if we order $a_1 \ge a_2 \ge a_3 \ge a_4$, we see $2(a_3 + a_4) \ge (a_1+a_2)+(a_3+a_4) = s_A \geq 0$, so $(a_3, a_4)$ isn't a couple that satisfies the conditions of the problem. Also, $2(a_4 + a_2) = (a_4 + a_4) + (a_2 + a_2) \ge (a_4+a_3)+(a_2+a_1) = s_A \ge 0$, so again $(a_2, a_4)$ isn't a good couple. We have in total 6 couples. So $n_A \leq 6-2=4$. We now find all sets $A$ with $n_A = 4$. If $(a,b)$ and $(c,d)$ are both good couples, and $A=\{a, b, c, d\}$, we have $a+b=c+d=s_A/2$. So WLOG $A=\{a,b,a+x,b-x\}$ with $x > 0$ and $a < b, b-x, a+x$. It's easy to see $a=a_1$ and since $(a_2, a_4),(a_3,a_4)$ are bad, all couples containing $a$ must be good. Obviously $(a,b)$ and $(a+x,b-x)$ are good ($s_A=2(a+b)$). So we have $2a+x | 2a+2b$ and $a+b-x|2a+2b \Rightarrow a+b-x|2x$. Using the second equation, we see that if $y=a+b$, $y-x|2x \Rightarrow yk_1-xk_1=2x \Rightarrow yk_1 = x(2+k_1) \Rightarrow y=x((2+k_1)/k_1)$, for some $k_1$ a positive integer. So now we use the first equation to get $2ak_2 + xk_2 = 2y = 2x(2+k_1)/k_1 \Rightarrow 2ak_2 = x(\frac{4+2k_1}{k_1}-k_2) \Rightarrow 2a=x(\frac{4+2k_1}{k_1k_2} - 1)$, for a natural $k_2$. Finally, we obtain $k_1 | 4+2k-1 \Rightarrow k_1 | 4 \Rightarrow k_1=$ 1, 2 or 4. We divide in cases: CASE I: $k_1=1$. So $y=3x$ and $2a=x((\frac{6}{k_2}) -1)$. But $a < b-x \Rightarrow 2a < y-x=2x \Rightarrow (6/k_2) - 1 < 2 \Rightarrow k_2 > 2 \Rightarrow k_2 =$3, 4,5 or 6. $k_2=6$ implies $a=0$, impossible. $a=x$ when $k_2=3$. We easily see $b=3x=3x$ and $A=\{x, 3x, 3x-x, 2x\}$, impossible since $3x-x=2x$. When $k_2=4$, $a=x/4=y/12$, and we get $\{11a, a, 5a, 7a\}$.Uf $k_2=5$, $a=x/5=y/15$ and we get $\{a, 14a, 6a, 9a\}$. CASE II and III:$k_1=$2, 4. Left to the reader. ANSWER: $\{11a, a, 5a, 7a\}$,$\{a, 11a, 19a, 29a\}$, for any positive integer $a$. (Note: The above solution looks generally correct, but the actual answer should be $\{11a, a, 5a, 7a\}$,$\{a, 11a, 19a, 29a\}$. You can check that $\{a, 14a, 6a, 9a\}$ doesn't actually work. -Someone who didn't write up the above solution but solved the problem in a similar way)
null
null
null
null
null
null
null
null
null
null
null
null
null
null
null
null
null
null
null
null
null
null
null
null
null
null
null
Given any set $A = \{a_1, a_2, a_3, a_4\}$ of four distinct positive integers, we denote the sum $a_1 +a_2 +a_3 +a_4$ by $s_A$. Let $n_A$ denote the number of pairs $(i, j)$ with $1 \leq i < j \leq 4$ for which $a_i +a_j$ divides $s_A$. Find all sets $A$ of four distinct positive integers which achieve the largest possible value of $n_A$. Author: Fernando Campos, Mexico
null
null
null
null
null
null
null
null
null
null
null
null
null
null
null
null
null
null
null
null
null
null
null
null
null
null
null
Same as Solution 2, except noticing that (letting $s = \dfrac{a + b + c}{2}$ be the semi-perimeter): --Suli 18:21, 8 February 2015 (EST)
null
null
null
null
null
null
As before in Solution 2, we find that $\angle{JFL} = \dfrac{\angle{A}}{2}.$ But it is clear that $AJ$ bisects $\angle{KAL}$, so $\angle{JAL} = \dfrac{\angle{A}}{2} = \angle{JFL}$ and hence $AFJL$ is cyclic. In particular, $\angle{AFJ} = \angle{ALJ} = 90^\circ$, and continue as in Solution 2.
null
Given triangle $ABC$ the point $J$ is the centre of the excircle opposite the vertex $A.$ This excircle is tangent to the side $BC$ at $M$, and to the lines $AB$ and $AC$ at $K$ and $L$, respectively. The lines $LM$ and $BJ$ meet at $F$, and the lines $KM$ and $CJ$ meet at $G.$ Let $S$ be the point of intersection of the lines $AF$ and $BC$, and let $T$ be the point of intersection of the lines $AG$ and $BC.$ Prove that $M$ is the midpoint of $ST$.
First, $BK = BM$ because $BK$ and $BM$ are both tangents from $B$ to the excircle $J$. Then $BJ \bot KM$. Call the $X$ the intersection between $BJ$ and $KM$. Similarly, let the intersection between the perpendicular line segments $CJ$ and $LM$ be $Y$. We have $\angle XBM = \angle XBK = \angle FBA$ and $\angle XMB = \angle XKB$. We then have, $\angle XBM + \angle XBK + \angle XMB + \angle XKB = \angle MBK + \angle XMB + \angle XKB$ $= \angle MBK + \angle KMB + \angle MKB = 180^{\circ}$. So $\angle XBM = 90^{\circ} - \angle XMB$. We also have $180^{\circ} = \angle FBA + \angle ABC + \angle XBM = 2\angle XBM + \angle ABC = 180^{\circ} - 2\angle XMB$ $+ \angle ABC$. Then $\angle ABC = 2\angle XMB$. Notice that $\angle XFM = 90^{\circ} - \angle XMB - \angle BMF = 90^{\circ} - \angle XMB - \angle YMC$. Then, $\angle ACB = 2\angle YMC$. $\angle BAC = 180^{\circ} - \angle ABC - \angle ACB = 180^{\circ} - 2(\angle XMB + \angle YMC)$ $= 2(90^{\circ} - (\angle XMB + \angle YMC) = 2\angle XFM$. Similarly, $\angle BAC = 2\angle YGM$. Draw the line segments $FK$ and $GL$. $\triangle FXK$ and $\triangle FXM$ are congruent and $\triangle GYL$ and $\triangle GYM$ are congruent. Quadrilateral $AFJL$ is cyclic because $\angle JAL = \frac{\angle BAC}{2} = \angle XFM = \angle JFL$. Quadrilateral $AFKJ$ is also cyclic because $\angle JAK = \frac{\angle BAC}{2} = \angle XFM = \angle XFK = \angle JFK$. The circumcircle of $\triangle AFJ$ also contains the points $K$ and $J$ because there is a circle around the quadrilaterals $AFJL$ and $AFKJ$. Therefore, pentagon $AFKJL$ is also cyclic. Finally, quadrilateral $AGLJ$ is cyclic because $\angle JAL = \frac{\angle BAC}{2} = \angle YGM = \angle YGL = \angle JGL$. Again, $\triangle AJL$ is common in both the cyclic pentagon $AFKJL$ and cyclic quadrilateral $AGLJ$, so the circumcircle of $\triangle AJL$ also contains the points $F$, $K$, and $G$. Therefore, hexagon $AFKJLG$ is cyclic. Since $\angle AKJ$ and $\angle ALJ$ are both right angles, $AJ$ is the diameter of the circle around cyclic hexagon $AFKJLG$. Therefore, $\angle AFJ$ and $\angle AGJ$ are both right angles. $\triangle BFS$ and $\triangle BFA$ are congruent by ASA congruency, and so are $\triangle CGT$ and $\triangle CGA$. We have $SB = AB$, $TC = AC$, $BM = BK$, and $CM = CL$. Since $AK$ and $AL$ are tangents from $A$ to the circle $J$, $AK = AL$. Then, we have $AK = AL$, which becomes $AB + BK = AC + CL$, which is $SB + BM = TC + CM$, or $SM = TM$. This means that $M$ is the midpoint of $ST$. QED --Aopsqwerty 21:19, 19 July 2012 (EDT)
For simplicity, let $A, B, C$ written alone denote the angles of triangle $ABC$, and $a$, $b$, $c$ denote its sides. Let $R$ be the radius of the A-excircle. Because $CM = CL$, we have $CML$ isosceles and so $\angle{CML} = \dfrac{\angle{C}}{2}$ by the Exterior Angle Theorem. Then because $\angle{FBS} = 90^\circ - \dfrac{B}{2}$, we have $\angle{BFM} = \dfrac{\angle{A}}{2}$, again by the Exterior Angle Theorem. Notice that $\angle{BJM} = \dfrac{\angle{B}}{2}$ and $\angle{CJM} = \dfrac{\angle{C}}{2}$, and so after converting tangents to sine and cosine. Thus, It follows that $BM = a \sin \dfrac{B}{2} \cos \dfrac{C}{2} \sec \frac{A}{2}$. By the Law of Sines on triangle $BFM$ and $ABC$ and the double-angle formula for sine, we have Therefore, triangle $BFA$ is congruent to a right triangle with hypotenuse length $c$ and one angle of measure $90^\circ - \dfrac{B}{2}$ by SAS Congruence, and so $\angle{BFA} = 90^\circ$. It then follows that triangles $BFS$ and $BFA$ are congruent by $ASA$, and so $AF = FS$. Thus, $J$ lies on the perpendicular bisector of $AS$. Similarly, $J$ lies on the perpendicular bisector of $AT$, and so $J$ is the circumcenter of $ATS$. In particular, $J$ lies on the perpendicular bisector of $ST$, and so, because $JM$ is perpendicular to $ST$, $M$ must be the midpoint of $ST$, as desired. --Suli 17:53, 8 February 2015 (EST)
null
null
null
null
null
null
null
null
null
null
null
null
null
null
null
null
null
null
null
null
null
null
null
null
null
null
null
null
null
null
null
null
null
null
Prove that for any pair of positive integers $k$ and $n$, there exist $k$ positive integers $m_1,m_2,...,m_k$ (not necessarily different) such that $1+\frac{2^k-1}{n}=(1+\frac{1}{m_1})(1+\frac{1}{m_2})...(1+\frac{1}{m_k})$.
We prove the claim by induction on $k$. Base case: If $k = 1$ then $1 +\frac{2^1-1}{n} = 1 + \frac{1}{n}$, so the claim is true for all positive integers $n$. Inductive hypothesis: Suppose that for some $m \in \mathbb{Z}^{+}$ the claim is true for $k = m$, for all $n \in \mathbb{Z}^{+}$. Inductive step: Let $n$ be arbitrary and fixed. Case on the parity of $n$: [Case 1: $n$ is even] $1 + \frac{2^{m+1} - 1}{n} = \left( 1 + \frac{2^{m} - 1}{\frac{n}{2}} \right) \cdot \left( 1 + \frac{1}{n + 2^{m+1} - 2} \right)$ [Case 2: $n$ is odd] $1 + \frac{2^{m+1} - 1}{n} = \left( 1 + \frac{2^{m}-1}{\frac{n+1}{2}} \right) \cdot \left( 1 + \frac{1}{n} \right)$ In either case, $1 + \frac{2^{m+1} - 1}{n} = \left( 1 + \frac{2^m - 1}{c} \right) \cdot \left( 1 + \frac{1}{a_{m+1}} \right)$ for some $c, a_{m+1} \in \mathbb{Z}^+$. By the induction hypothesis we can choose $a_1, ..., a_m$ such that $\left( 1 + \frac{2^m - 1}{c} \right) = \prod_{i=1}^{m} (1 + \frac{1}{a_i})$. Therefore, since $n$ was arbitrary, the claim is true for $k = m+1$, for all $n$. Our induction is complete and the claim is true for all positive integers $n$, $k$.
null
null
null
null
null
null
null
null
null
null
null
null
null
null
null
null
null
null
null
null
null
null
null
null
null
null
null
null
null
null
null
null
null
null
null
Let $a_0<a_1<a_2<\cdots \quad$ be an infinite sequence of positive integers, Prove that there exists a unique integer $n\ge1$ such that
Define $f(n) = a_0 + a_1 + \dots + a_n - n a_{n+1}$. (In particular, $f(0) = a_0.$) Notice that because $a_{n+2} \ge a_{n+1}$, we have Thus, $f(n) > f(n+1)$; i.e., $f$ is monotonic decreasing. Therefore, because $f(0) > 0$, there exists a unique $N$ such that $f(N-1) > 0 \ge f(N)$. In other words, This rearranges to give Define $g(n) = a_0 + a_1 + \dots + a_n - n a_n$. Then because $a_{n+1} > a_n$, we have Therefore, $g$ is also monotonic decreasing. Note that $g(N+1) = a_0 + a_1 + \dots + a_{N+1} - (N+1) a_{N+1} \le 0$ from our inequality, and so $g(k) \le 0$ for all $k > N$. Thus, the given inequality, which requires that $g(n) > 0$, cannot be satisfied for $n > N$, and so $N$ is the unique solution to this inequality. --Suli 22:38, 7 February 2015 (EST)
null
null
null
null
null
null
null
null
null
null
null
null
null
null
null
null
null
null
null
null
null
null
null
null
null
null
null
null
null
null
null
null
null
null
null
We say that a finite set $\mathcal{S}$ in the plane is balanced if, for any two different points $A$, $B$ in $\mathcal{S}$, there is a point $C$ in $\mathcal{S}$ such that $AC=BC$. We say that $\mathcal{S}$ is centre-free if for any three points $A$, $B$, $C$ in $\mathcal{S}$, there is no point $P$ in $\mathcal{S}$ such that $PA=PB=PC$.
Part (a): We explicitly construct the sets $\mathcal{S}$. For odd $n$, $\mathcal{S}$ can be taken to be the vertices of regular polygons $P_n$ with $n$ sides: given any two vertices $A$ and $B$, one of the two open half-spaces into which $AB$ divides $P_n$ contains an odd number of $k$ of vertices of $P_n$. The $((k+1)/2)^{th}$ vertex encountered while moving from $A$ to $B$ along the circumcircle of $P_n$ is therefore equidistant from $A$ and $B$. If $n \geq 4$ is even, choose $m\geq 0$ to be the largest integer such that Hence $x < 4\pi/3 < 2\pi$. Consider a circle $K$ with centre $O$, and let $A_1, \ldots, A_{n-1}$ be distinct points placed counterclockwise (say) on $K$ such that $\angle A_iOA_{i+1}=\pi/3/2^m$ (for $i=1,\ldots,n-2$). Hence for any line $OA_i$, there is a line $OA_j$ such that $\angle A_iOA_j=\pi/3$ (using the facts that $2\pi > x=\angle A_1OA_{n-1} \geq 2\pi/3$, and $n-1$ odd). Thus $O$, $A_i$ and $A_j$ form an equilateral triangle. In other words, for arbitrary $A_i$, there exists $A_j$ equidistant to $O$ and $A_i$. Also given any $i,j$ such that $1 \leq i, j \leq n-1$, $O$ is equidistant to $A_i$ and $A_j$. Hence the $n$ points $O, A_1, \ldots, A_{n-1}$ form a balanced set. Part (b): Note that if $n$ is odd, the set $\mathcal{S}$ of vertices of a regular polygon $P_n$ of $n$ sides forms a balanced set (as above) and a centre-free set (trivially, since the centre of the circumscribing circle of $P_n$ does not belong to $\mathcal{S}$). For $n$ even, we prove that a balanced, centre free set consisting of $n$ points does not exist. Assume that $\mathcal{S}=\{A_i: 1\leq i \leq n\}$ is centre-free. Pick an arbitrary $A_i \in \mathcal{S}$, and let $n_i$ be the number of distinct non-ordered pairs of points $(A_j,A_k)$ ($j\neq k$) to which $A_i$ is equidistant. Any two such pairs are disjoint (for, if there were two such pairs $(A_r,A_s)$ and $(A_r, A_t)$ with $r, s, t$ distinct, then $A_i$ would be equidistant to $A_r$, $A_s$, and $A_t$, violating the centre-free property). Hence $n_i \leq (n-2)/2$ (we use the fact that $n$ is even here), which means $\sum_i n_i \leq n(n-2)/2 = n(n-1)/2 -n/2$. Hence there are at least $n/2$ non-ordered pairs $(A_j, A_k)$ such that no point in $\mathcal{S}$ is equidistant to $A_j$ and $A_k$.
null
null
null
null
null
null
null
null
null
null
null
null
null
null
null
null
null
null
null
null
null
null
null
null
null
null
null
null
null
null
null
null
null
null
null
Triangle $BCF$ has a right angle at $B$. Let $A$ be the point on line $CF$ such that $FA=FB$ and $F$ lies between $A$ and $C$. Point $D$ is chosen so that $DA=DC$ and $AC$ is the bisector of $\angle{DAB}$. Point $E$ is chosen so that $EA=ED$ and $AD$ is the bisector of $\angle{EAC}$. Let $M$ be the midpoint of $CF$. Let $X$ be the point such that $AMXE$ is a parallelogram. Prove that $BD,FX$ and $ME$ are concurrent.
This problem needs a solution. If you have a solution for it, please help us out by adding it.
null
null
null
null
null
null
null
null
null
null
null
null
null
null
null
null
null
null
null
null
null
null
null
null
null
null
null
null
null
null
null
null
null
null
null
For each integer $a_0 > 1$, define the sequence $a_0, a_1, a_2, \ldots$ for $n \geq 0$ as Determine all values of $a_0$ such that there exists a number $A$ such that $a_n = A$ for infinitely many values of $n$.
First we observe the following: When we start with $a_0=3$, we get $a_1=6$, $a_2=9$, $a_3=3$ and the pattern $3,6,9$ repeats. When we start with $a_0=6$, we get $a_1=9$, $a_2=3$, $a_3=6$ and the pattern $3,6,9$ repeats. When we start with $a_0=9$, we get $a_1=3$, $a_2=6$, $a_3=9$ and the pattern $3,6,9$ repeats. When we start with $a_0=12$, we get $a_1=15$, $a_2=15$,..., $a_8=36$, $a_9=6$, $a_{10}=9$, $a_{11}=3$ and the pattern $3,6,9$ repeats. When this pattern $3,6,9$ repeats, this means that there exists a number $A$ such that $a_n = A$ for infinitely many values of $n$ and that number $A$ is either $3,6,$ or $9$. When we start with any number $a_0\not\equiv 0\; mod\; 3$, we don't see a repeating pattern. Therefore the claim is that $a_0=3k$ where $k$ is a positive integer and we need to prove this claim. When we start with $a_0=3k$, the next term if it is not a square is $3k+3$, then $3k+6$ and so on until we get $3k+3p$ where $p$ is an integer and $(k+p)=3q^2$ where $q$ is an integer. Then the next term will be $\sqrt{9q^2}=3q$ and the pattern repeats again when $q=k$ or when $q=3$ or $6$. In order for these patterns to repeat, any square in the sequence need to be a multiple of 3. To try the other two cases where $a_0\not\equiv 0\; mod\; 3$, we can try $a_0=3k\pm 1$ then the next terms will be in the form $3k+3p\pm 1 = 3(k+p) \pm 1$. When $3(k+p) \pm 1$ is a square, it will not be a multiple of $3$ because $3(k+p) \pm 1$ is not a multiple of $3$ and $3(k+p) \pm 1 \ne 9q^2$ because $3(k+p) \pm 1 \equiv \pm 1\; mod\; 3$ and $q^2$ would have to be $\frac{(k+p)}{3} \pm \frac{1}{9}$ which is not an integer even if $k+p$ is a multiple of $3$. Therefore the pattern doesn't repeat for any of the other cases where $a_0=3k\pm 1$ and only repeats when $a_0\equiv 0\; mod\; 3$ So, the answer to this problem is $a_0=3k\;\forall k \in \mathbb{Z}^{+}$ and $A=3,6,$ or $9$. ~Tomas Diaz. orders@tomasdiaz.com Alternate solutions are always welcome. If you have a different, elegant solution to this problem, please add it to this page.
null
null
null
null
null
null
null
null
null
null
null
null
null
null
null
null
null
null
null
null
null
null
null
null
null
null
null
null
null
null
null
null
null
null
null
Let $\Gamma$ be the circumcircle of acute triangle $ABC$. Points $D$ and $E$ are on segments $AB$ and $AC$ respectively such that $AD = AE$. The perpendicular bisectors of $BD$ and $CE$ intersect minor arcs $AB$ and $AC$ of $\Gamma$ at points $F$ and $G$ respectively. Prove that lines $DE$ and $FG$ are either parallel or they are the same line.
http://wiki-images.artofproblemsolving.com/5/5d/FB_IMG_1531446409131.jpg The diagram is certainly not to scale, but the argument is sound (I believe) and involves re-ordering the construction as specified in the original problem so that an identical state of affairs results, yet in so doing differently it is made clear that the line segments in question are parallel. Construct a right-angled triangle ABC'. Select an arbitrary point H along the segment BC', and from point H select an arbitrary point F such that the segment HF is perpendicular to the segment AB. Mark the distance from the intersection of HF and AB to B at B" (i.e., HF is a perpendicular bisector). It follows that the triangle B"FB is isosceles. Construct an isosceles triangle FHG. Mark the distance of AB" along AC' at C". (From here, a circle can be constructed according to the sets of points A, B, F, and A, B, G. Points F and G may be repositioned to allow for these circles to coincide; also, point H may be repositioned so that point C falls on the coinciding circle, understood that HG is the other perpendicular bisector.) Assign the angle BAC the value α. Hence, the angle FHG has the value 180°-α, and the angle HFG (also, HGF) has the value α/2. Assign the angle BFH the value β. Hence, the angle B'FB" has the value β-α/2. Consequently, the angle FB'B" has the value 180°-(β-α/2)-(90°-β) = 90°+α/2, and so too its vertical angle BB'G. As the triangle B"AC" is isosceles, and its subtended angle has the value α, the angles BB"C" and CC"B" both have the value 90°+α/2. It follows therefore that segments B"C" and FG are parallel. (N.B. Points D and E, as given in the wording of the original problem, have been renamed B" and C" here.)
The essence of the proof is using a rhombus formed by the perpendicular bisectors of the segments $BD, CE, AB$ and $AC$ and the parallelism of its diagonal and the base of the triangle formed by the perpendiculars from one vertex of the rhombus. The perpendicular bisectors of the segments $BD, CE, AB$ and $AC$ intersect at points $O$ (the circumcenter $ABC), H, H',$ and $Q.$ Let $AD = AE = 2x, AC = 2b.$ The distance from the line $HO$ to point $C$ is $b,$ from $QH'$ to $C$ is $b - x.$ Therefore, the distance between lines $HO$ and $QH'$ is $x.$ Similarly, the distance between the lines $HQ$ and $OH'$ is $x.$ The quadrilateral $OHQH'$ is formed by the intersection of two pairs of equidistant lines $\implies$ $OHQH'$ is a parallelogram with equal heights $\implies$ $OHQH'$ is a rhombus. Let $I$ and $I'$ be the feet of the heights of the rhombus from the vertex $O.$ In isosceles triangles $\triangle ADE$ and $\triangle OII'$ the sides are parallel, hence the bases of these triangles are parallel, $DE||II'.$ The feet of the heights $I$ and $I'$ divide the sides of the rhombus in the same ratio, which means that the diagonal $HH'$ of the rhombus is parallel to the segments $II' || DE.$ The angles between the diagonal of the rhombus and its sides are the same, so $\angle HQO = \angle H'QO.$ The lines $QF$ and $QG$ are symmetrical with respect to the diameter $OQ,$ so $QF = QG, QF' = QG'.$ The homothety of triangles $QHH', QFG,$ and $QF'G'$ centered at $Q$ implies $FG || F'G' || HH'.$ Therefore, these three lines are parallel to $DE.$ Note that triangle $ABC$ may be obtuse. vladimir.shelomovskii@gmail.com, vvsss
null
null
null
null
null
null
Let us substitute $0$ in for $a$ to get Now, since the domain and range of $f$ are the same, we can let $x = f(b)$ and $f(0)$ equal some constant $c$ to get Therefore, we have found that all solutions must be of the form $f(x) = 2x + c.$ Plugging back into the original equation, we have: $4a + c + 4b + 2c = 4a + 4b + 2c + c$ which is true. Therefore, we know that $f(x) = 2x + c$ satisfies the above for any integral constant c, and that this family of equations is unique. (This solution does not work though because we don't know that $f$ is surjective)
null
null
null
null
null
null
null
null
null
null
null
null
null
null
null
null
null
null
null
null
null
null
null
null
null
null
null
Let $\mathbb{Z}$ be the set of integers. Determine all functions $f : \mathbb{Z} \to \mathbb{Z}$ such that, for all integers $a$ and $b$,
null
We plug in $a=-b=x$ and $a=-b=x+k$ to get respectively. Setting them equal to each other, we have the equation and moving "like terms" to one side of the equation yields Seeing that this is a difference of outputs of $f,$ we can relate this to slope by dividing by $2k$ on both sides. This gives us which means that $f$ is linear. (Functional equations don't work like that unfortunately) Let $f(x)=mx+n.$ Plugging our expression into our original equation yields $2ma+2mb+3n=m^2a+m^2b+mn+n,$ and letting $b$ be constant, this can only be true if $2m=m^2 \implies m=0,2.$ If $m=0,$ then $n=0,$ which implies $f(x)=0.$ However, the output is then not all integers, so this doesn't work. If $m=2,$ we have $f(x)=2x+n.$ Plugging this in works, so the answer is $f(x)=2x+c$ for some integer $c.$
null
null
null
null
null
null
null
null
null
null
null
null
null
null
null
null
null
null
null
null
null
null
null
null
null
null
null
null
null
null
null
null
null
null
Consider the convex quadrilateral $ABCD$. The point $P$ is in the interior of $ABCD$. The following ratio equalities hold: Prove that the following three lines meet in a point: the internal bisectors of angles $\angle ADP$ and $\angle PCB$ and the perpendicular bisector of segment $\overline{AB}$.
null
null
null
null
null
null
null
null
If we can guarantee that there exist $3$ cards such that every pair of them sum to a perfect square, then we can guarantee that one of the piles contains $2$ cards that sum to a perfect square. Assume the perfect squares $p^2$, $q^2$, and $r^2$ satisfy the following system of equations: where $a$, $b$, and $c$ are numbers on three of the cards. Solving for $a$, $b$, and $c$ in terms of $p$, $q$, and $r$ tells us that $a = \frac{p^2 + r^2 - q^2}{2}$, $b=\frac{p^2 + q^2 - r^2}{2}$, and $c=\frac{q^2 + r^2 - p^2}{2}$. We can then substitute $p^2 = (2e-1)^2$, $q^2 = (2e)^2$, and $r^2 = (2e+1)^2$ to cancel out the $2$s in the denominatior, and simplifying gives $a = 2e^2 + 1$, $b = 2e(e-2)$, and $c = 2e(e+2)$. Now, we have to prove that there exists three numbers in these forms between $n$ and $2n$ when $n \ge 100$. Notice that $b$ will always be the least of the three and $c$ will always be the greatest of the three. So it is sufficient to prove that there exists numbers in the form $2e(e-2)$ and $2e(e+2)$ between $n$ and $2n$. For two numbers in the form of $2e(e-2)$ and $2e(e+2)$ to be between $n$ and $2n$, the inequalities must be satisfied. We can then expand and simplify to get that Then, we can complete the square on the left sides of both inequalities and isolate $e$ to get that Notice that $e$ must be an integer, so there must be an integer between $\sqrt{1 + n} - 1$ and $\sqrt{1 + \frac{n}{2}} + 1$. If $\sqrt{1 + n} - 1$ and $\sqrt{1 + \frac{n}{2}} + 1$ differ by at least $1$, then we can guarantee that there is an integer between them (and those integers are the possible values of $e$). Setting up the inequality $\sqrt{1 + n} - \sqrt{1 + \frac{n}{2}} - 2 \ge 1$ and solving for $n$ tells us that $n \in [107, \infty)$ always works. Testing the remaining $7$ numbers ($100$ to $106$) manually tells us that there is an integer between $\sqrt{1 + n} - 1$ and $\sqrt{1 + \frac{n}{2}} + 1$ when $n \ge 100$. Therefore, there exists a triplet of integers $(a,b,c)$ with $a, b, c \in \{n, n+1, ..., 2n\}$ when $n \ge 100$ such that every pair of the numbers sum to a perfect square. By the pigeonhole principle, we know that $2$ of the numbers must be on cards in the same pile, and hence, when $n \ge 100$, there will always be a pile with $2$ numbers that sum to a perfect square. $\square$ ~Mathdreams
null
null
null
null
null
null
null
null
null
null
null
null
null
null
null
null
null
null
null
null
null
null
null
null
null
null
null
Let $n \geq 100$ be an integer. Ivan writes the numbers $n, n+1, \ldots, 2 n$ each on different cards. He then shuffles these $n+1$ cards, and divides them into two piles. Prove that at least one of the piles contains two cards such that the sum of their numbers is a perfect square.
null
For the statement to be true, there must be at least three pairs whose sum is each a perfect square. There must be p,q,r such that p+q = x^2 and q+r = y^2, p+r = z^2. by equation 1 if we add (2) and (3) to (1), (1) + (2) + (3) => At this time 100 ≤ n, so let's put n = 100 to this where x = 16, y = 18, z = 20 fits perfectly therefore the minimum of n fits the proposition so the proposition is true ~Mathhyhyhy
null
null
null
null
null
null
null
null
null
null
null
null
null
null
null
null
null
null
null
null
null
null
null
null
null
null
null
null
null
null
null
null
null
null
The Bank of Oslo issues two types of coin: aluminium (denoted A) and bronze (denoted B). Marianne has $n$ aluminium coins and $n$ bronze coins, arranged in a row in some arbitrary initial order. A chain is any subsequence of consecutive coins of the same type. Given a fixed positive integer $k\le 2n$, Marianne repeatedly performs the following operation: she identifies the longest chain containing the $k^{th}$ coin from the left, and moves all coins in that chain to the left end of the row. For example, if $n = 4$ and $k = 4$, the process starting from the ordering AABBBABA would be AABBBABA → BBBAAABA → AAABBBBA → BBBBAAAA → BBBBAAAA → ... Find all pairs $(n, k)$ with $1 \le k \le 2n$ such that for every initial ordering, at some moment during the process, the leftmost $n$ coins will all be of the same type.
https://www.youtube.com/watch?v=nYD-qIOdi_c [Video contains solutions to all day 1 problems] https://youtu.be/KHn3xD6wS3A [Video contains problem 1 discussion] We call a chain basic when it is the largest possible for the coins it consists of. Let \(A=[i,j]\) be the basic chain with the \(i\)-th and \(j\)-th coins being the first and last, respectively. Claim: Proof: For \(k < n\), it is easy to see that the arrangement \(A\ldots AB\ldots BA\) remains the same. For \(k > \lceil \frac{3n}{2} \rceil = 2n - \lfloor \frac{n}{2} \rfloor\), we obtain the arrangement \(A\ldots AB\ldots BA\ldots AB\ldots B\), where each basic chain consists of \(\lfloor \frac{n}{2} \rfloor, \lceil \frac{n}{2} \rceil, \lceil \frac{n}{2} \rceil, \lfloor \frac{n}{2} \rfloor\) coins, respectively. Since the number of coins in the last chain is \(\geq \lfloor \frac{n}{2} \rfloor\), it follows that \(k\) is greater than the number of the remaining coins, or in other words, it is always contained in the last chain. However, we have a loop: We will prove that in any other case, the number of basic chains decreases by a constant, which proves the claim. For \(k \in B=[l, m]\), where \(l > 1\) and \(m < 2n\), the basic chains \(B_1=[l{'}, l-1]\) and \(B_2=[m+1, m']\) merge into one, and we are done since it is impossible to increase. For \(k \in C=[1, l]\), where \(n + 1 > l \geq k \geq n\), it holds that \(l = n\), which is what we need to prove. For \(k \in D=[m, 2n]\), we will prove that the basic chains are of quantity 2 or 3 (two are obtained with one move): Indeed, if there are at least 4 basic chains, from the beginning of the pigeonhole, we have at least one chain with a number of coins < \(\lfloor \frac{n}{2} \rfloor + 1 \leq 2n - k + 1\). Therefore, \(k\) does not belong to this chain when it is the last, and then the number of basic chains decreases, which completes the proof. $\blacksquare$ In conclusion, such pairs are \((n, k)\), where \(k \in \{n, n+1, \ldots, \lceil \frac{3n}{2} \rceil\}\).
null
null
null
null
End of preview.

No dataset card yet

New: Create and edit this dataset card directly on the website!

Contribute a Dataset Card
Downloads last month
5